Programs & Examples On #Qif

Safe Area of Xcode 9

Apple introduced the topLayoutGuide and bottomLayoutGuide as properties of UIViewController way back in iOS 7. They allowed you to create constraints to keep your content from being hidden by UIKit bars like the status, navigation or tab bar. These layout guides are deprecated in iOS 11 and replaced by a single safe area layout guide.

Refer link for more information.

Why does C++ code for testing the Collatz conjecture run faster than hand-written assembly?

C++ programs are translated to assembly programs during the generation of machine code from the source code. It would be virtually wrong to say assembly is slower than C++. Moreover, the binary code generated differs from compiler to compiler. So a smart C++ compiler may produce binary code more optimal and efficient than a dumb assembler's code.

However I believe your profiling methodology has certain flaws. The following are general guidelines for profiling:

  1. Make sure your system is in its normal/idle state. Stop all running processes (applications) that you started or that use CPU intensively (or poll over the network).
  2. Your datasize must be greater in size.
  3. Your test must run for something more than 5-10 seconds.
  4. Do not rely on just one sample. Perform your test N times. Collect results and calculate the mean or median of the result.

BeautifulSoup getText from between <p>, not picking up subsequent paragraphs

You are getting close!

# Find all of the text between paragraph tags and strip out the html
page = soup.find('p').getText()

Using find (as you've noticed) stops after finding one result. You need find_all if you want all the paragraphs. If the pages are formatted consistently ( just looked over one), you could also use something like

soup.find('div',{'id':'ctl00_PlaceHolderMain_RichHtmlField1__ControlWrapper_RichHtmlField'})

to zero in on the body of the article.

Creating a random string with A-Z and 0-9 in Java

You can easily do that with a for loop,

public static void main(String[] args) {
  String aToZ="ABCD.....1234"; // 36 letter.
  String randomStr=generateRandom(aToZ);

}

private static String generateRandom(String aToZ) {
    Random rand=new Random();
    StringBuilder res=new StringBuilder();
    for (int i = 0; i < 17; i++) {
       int randIndex=rand.nextInt(aToZ.length()); 
       res.append(aToZ.charAt(randIndex));            
    }
    return res.toString();
}

PHP: Best way to check if input is a valid number?

The most secure way

if(preg_replace('/^(\-){0,1}[0-9]+(\.[0-9]+){0,1}/', '', $value) == ""){
  //if all made of numbers "-" or ".", then yes is number;
}

'cout' was not declared in this scope

Use std::cout, since cout is defined within the std namespace. Alternatively, add a using std::cout; directive.

How to run Nginx within a Docker container without halting?

nginx, like all well-behaved programs, can be configured not to self-daemonize.

Use the daemon off configuration directive described in http://wiki.nginx.org/CoreModule.

C++ cout hex values?

Use std::uppercase and std::hex to format integer variable a to be displayed in hexadecimal format.

#include <iostream>
int main() {
   int a = 255;

   // Formatting Integer
   std::cout << std::uppercase << std::hex << a << std::endl; // Output: FF
   std::cout << std::showbase  << std::hex << a << std::endl; // Output: 0XFF
   std::cout << std::nouppercase << std::showbase  << std::hex << a << std::endl; // Output: 0xff

   return 0;
}

Calculate mean and standard deviation from a vector of samples in C++ using Boost

Using accumulators is the way to compute means and standard deviations in Boost.

accumulator_set<double, stats<tag::variance> > acc;
for_each(a_vec.begin(), a_vec.end(), bind<void>(ref(acc), _1));

cout << mean(acc) << endl;
cout << sqrt(variance(acc)) << endl;

 

How to parse JSON to receive a Date object in JavaScript?

Dates are always a nightmare. Answering your old question, perhaps this is the most elegant way:

eval(("new " + "/Date(1455418800000)/").replace(/\//g,""))

With eval we convert our string to javascript code. Then we remove the "/", into the replace function is a regular expression. As we start with new then our sentences will excecute this:

new Date(1455418800000)

Now, one thing I started using long time ago, is long values that are represented in ticks... why? well, localization and stop thinking in how is date configured in every server or every client. In fact, I use it too in databases.

Perhaps is quite late for this answer, but can help anybody arround here.

How do I change the text size in a label widget, python tkinter

Try passing width=200 as additional paramater when creating the Label.

This should work in creating label with specified width.

If you want to change it later, you can use:

label.config(width=200)

As you want to change the size of font itself you can try:

label.config(font=("Courier", 44))

Modify tick label text

I noticed that all the solutions posted here that use set_xticklabels() are not preserving the offset, which is a scaling factor applied to the ticks values to create better-looking tick labels. For instance, if the ticks are on the order of 0.00001 (1e-5), matplotlib will automatically add a scaling factor (or offset) of 1e-5, so the resultant tick labels may end up as 1 2 3 4, rather than 1e-5 2e-5 3e-5 4e-5.

Below gives an example:

The x array is np.array([1, 2, 3, 4])/1e6, and y is y=x**2. So both are very small values.

Left column: manually change the 1st and 3rd labels, as suggested by @Joe Kington. Note that the offset is lost.

Mid column: similar as @iipr suggested, using a FuncFormatter.

Right column: My suggested offset-preserving solution.

Figure here: enter image description here

Complete code here:

import matplotlib.pyplot as plt
import numpy as np

# create some *small* data to plot
x = np.arange(5)/1e6
y = x**2

fig, axes = plt.subplots(1, 3, figsize=(10,6))

#------------------The set_xticklabels() solution------------------
ax1 = axes[0]
ax1.plot(x, y)
fig.canvas.draw()
labels = [item.get_text() for item in ax1.get_xticklabels()]

# Modify specific labels
labels[1] = 'Testing'
labels[3] = 'Testing2'
ax1.set_xticklabels(labels)
ax1.set_title('set_xticklabels()')

#--------------FuncFormatter solution--------------
import matplotlib.ticker as mticker

def update_ticks(x, pos):
    if pos==1:
        return 'testing'
    elif pos==3:
        return 'testing2'
    else:
        return x

ax2=axes[1]
ax2.plot(x,y)
ax2.xaxis.set_major_formatter(mticker.FuncFormatter(update_ticks))
ax2.set_title('Func Formatter')

#-------------------My solution-------------------
def changeLabels(axis, pos, newlabels):
    '''Change specific x/y tick labels

    Args:
        axis (Axis): .xaxis or .yaxis obj.
        pos (list): indices for labels to change.
        newlabels (list): new labels corresponding to indices in <pos>.
    '''

    if len(pos) != len(newlabels):
        raise Exception("Length of <pos> doesn't equal that of <newlabels>.")

    ticks = axis.get_majorticklocs()
    # get the default tick formatter
    formatter = axis.get_major_formatter()
    # format the ticks into strings
    labels = formatter.format_ticks(ticks)

    # Modify specific labels
    for pii, lii in zip(pos, newlabels):
        labels[pii] = lii

    # Update the ticks and ticklabels. Order is important here.
    # Need to first get the offset (1e-6 in this case):
    offset = formatter.get_offset()
    # Then set the modified labels:
    axis.set_ticklabels(labels)
    # In doing so, matplotlib creates a new FixedFormatter and sets it to the xaxis
    # and the new FixedFormatter has no offset. So we need to query the
    # formatter again and re-assign the offset:
    axis.get_major_formatter().set_offset_string(offset)

    return

ax3 = axes[2]
ax3.plot(x, y)

changeLabels(ax3.xaxis, [1, 3], ['Testing', 'Testing2'])
ax3.set_title('With offset')

fig.show()
plt.savefig('tick_labels.png')

Caveat: it appears that solutions that use set_xticklabels(), including my own, relies on FixedFormatter, which is static and doesn't respond to figure resizing. To observe the effect, change the figure to a smaller size, e.g. fig, axes = plt.subplots(1, 3, figsize=(6,6)) and enlarge the figure window. You will notice that that only the mid column responds to resizing and adds more ticks as the figure gets larger. The left and right column will have empty tick labels (see figure below).

Caveat 2: I also noticed that if your tick values are floats, calling set_xticklabels(ticks) directly might give you ugly-looking strings, like 1.499999999998 instead of 1.5.

enter image description here

Concatenate strings from several rows using Pandas groupby

we can groupby the 'name' and 'month' columns, then call agg() functions of Panda’s DataFrame objects.

The aggregation functionality provided by the agg() function allows multiple statistics to be calculated per group in one calculation.

df.groupby(['name', 'month'], as_index = False).agg({'text': ' '.join})

enter image description here

How do I fix an "Invalid license data. Reinstall is required." error in Visual C# 2010 Express?

I had this problem and finally got passed it. I tried the solutions above to no effect. (I set my license keys to open permissions, set my clock forward, etc.) After two days I gave up...

In the end, I installed VS 2012 Express, which could handle VS 2010 solutions but could not compile 2010 code (without a COFF error). After finding this article on requiring VS 2010 to be installed to compile 2010 solution in VS 2012, I reinstalled VS 2010 even though I assumed it wouldn't work on its own. I tried opening 2010 anyway and it worked!

SVN- How to commit multiple files in a single shot

I've had no issues committing a few files like this:

svn commit fileDir1/ fileDir2/ -m "updated!"

How do I hide the bullets on my list for the sidebar?

You have a selector ul on line 252 which is setting list-style: square outside none (a square bullet). You'll have to change it to list-style: none or just remove the line.

If you only want to remove the bullets from that specific instance, you can use the specific selector for that list and its items as follows:

ul#groups-list.items-list { list-style: none }

Can a relative sitemap url be used in a robots.txt?

Good technical & logical question my dear friend. No in robots.txt file you can't go with relative URL of the sitemap; you need to go with the complete URL of the sitemap.

It's better to go with "sitemap: https://www.example.com/sitemap_index.xml"

In the above URL after the colon gives space. I also like to support Deepak.

TypeScript for ... of with index / key?

"Old school javascript" to the rescue (for those who aren't familiar/in love of functional programming)

for (let i = 0; i < someArray.length ; i++) {
  let item = someArray[i];
}

how to get the selected index of a drop down

If you are actually looking for the index number (and not the value) of the selected option then it would be

document.forms[0].elements["CCards"].selectedIndex 
/* You may need to change document.forms[0] to reference the correct form */

or using jQuery

$('select[name="CCards"]')[0].selectedIndex 

Identify if a string is a number

If you want to catch a broader spectrum of numbers, à la PHP's is_numeric, you can use the following:

// From PHP documentation for is_numeric
// (http://php.net/manual/en/function.is-numeric.php)

// Finds whether the given variable is numeric.

// Numeric strings consist of optional sign, any number of digits, optional decimal part and optional
// exponential part. Thus +0123.45e6 is a valid numeric value.

// Hexadecimal (e.g. 0xf4c3b00c), Binary (e.g. 0b10100111001), Octal (e.g. 0777) notation is allowed too but
// only without sign, decimal and exponential part.
static readonly Regex _isNumericRegex =
    new Regex(  "^(" +
                /*Hex*/ @"0x[0-9a-f]+"  + "|" +
                /*Bin*/ @"0b[01]+"      + "|" + 
                /*Oct*/ @"0[0-7]*"      + "|" +
                /*Dec*/ @"((?!0)|[-+]|(?=0+\.))(\d*\.)?\d+(e\d+)?" + 
                ")$" );
static bool IsNumeric( string value )
{
    return _isNumericRegex.IsMatch( value );
}

Unit Test:

static void IsNumericTest()
{
    string[] l_unitTests = new string[] { 
        "123",      /* TRUE */
        "abc",      /* FALSE */
        "12.3",     /* TRUE */
        "+12.3",    /* TRUE */
        "-12.3",    /* TRUE */
        "1.23e2",   /* TRUE */
        "-1e23",    /* TRUE */
        "1.2ef",    /* FALSE */
        "0x0",      /* TRUE */
        "0xfff",    /* TRUE */
        "0xf1f",    /* TRUE */
        "0xf1g",    /* FALSE */
        "0123",     /* TRUE */
        "0999",     /* FALSE (not octal) */
        "+0999",    /* TRUE (forced decimal) */
        "0b0101",   /* TRUE */
        "0b0102"    /* FALSE */
    };

    foreach ( string l_unitTest in l_unitTests )
        Console.WriteLine( l_unitTest + " => " + IsNumeric( l_unitTest ).ToString() );

    Console.ReadKey( true );
}

Keep in mind that just because a value is numeric doesn't mean it can be converted to a numeric type. For example, "999999999999999999999999999999.9999999999" is a perfeclty valid numeric value, but it won't fit into a .NET numeric type (not one defined in the standard library, that is).

How to send email to multiple address using System.Net.Mail

My code to solve this problem:

private void sendMail()
{   
    //This list can be a parameter of metothd
    List<MailAddress> lst = new List<MailAddress>();

    lst.Add(new MailAddress("[email protected]"));
    lst.Add(new MailAddress("[email protected]"));
    lst.Add(new MailAddress("[email protected]"));
    lst.Add(new MailAddress("[email protected]"));


    try
    {


        MailMessage objeto_mail = new MailMessage();
        SmtpClient client = new SmtpClient();
        client.Port = 25;
        client.Host = "10.15.130.28"; //or SMTP name
        client.Timeout = 10000;
        client.DeliveryMethod = SmtpDeliveryMethod.Network;
        client.UseDefaultCredentials = false;
        client.Credentials = new System.Net.NetworkCredential("[email protected]", "password");
        objeto_mail.From = new MailAddress("[email protected]");

        //add each email adress
        foreach (MailAddress m in lst)
        {
            objeto_mail.To.Add(m);
        }


        objeto_mail.Subject = "Sending mail test";
        objeto_mail.Body = "Functional test for automatic mail :-)";
        client.Send(objeto_mail);


    }
    catch (Exception ex)
    {
        MessageBox.Show(ex.Message);
    }
}

When should we use mutex and when should we use semaphore

A mutex is a special case of a semaphore. A semaphore allows several threads to go into the critical section. When creating a semaphore you define how may threads are allowed in the critical section. Of course your code must be able to handle several accesses to this critical section.

ModalPopupExtender OK Button click event not firing?

It could also be that the button needs to have CausesValidation="false". That worked for me.

Set space between divs

Float them both the same way and add the margin of 40px. If you have 2 elements floating opposite ways you will have much less control and the containing element will determine how far apart they are.

#left{
    float: left;
    margin-right: 40px;
}
#right{
   float: left;
}

NSAttributedString add text alignment

Xamarin.iOS

NSMutableParagraphStyle paragraphStyle = new NSMutableParagraphStyle();
paragraphStyle.HyphenationFactor = 1.0f;
var hyphenAttribute = new UIStringAttributes();
hyphenAttribute.ParagraphStyle = paragraphStyle;
var attributedString = new NSAttributedString(str: name, attributes: hyphenAttribute);

Select a date from date picker using Selenium webdriver

I tried this code, it may work for you also:

            DateFormat dateFormat2 = new SimpleDateFormat("dd"); 
            Date date2 = new Date();

            String today = dateFormat2.format(date2); 

            //find the calendar
            WebElement dateWidget = driver.findElement(By.id("dp-calendar"));  
            List<WebElement> columns=dateWidget.findElements(By.tagName("td"));  

            //comparing the text of cell with today's date and clicking it.
            for (WebElement cell : columns)
            {
               if (cell.getText().equals(today))
               {
                  cell.click();
                  break;
               }
            }

Unpacking a list / tuple of pairs into two lists / tuples

>>> source_list = ('1','a'),('2','b'),('3','c'),('4','d')
>>> list1, list2 = zip(*source_list)
>>> list1
('1', '2', '3', '4')
>>> list2
('a', 'b', 'c', 'd')

Edit: Note that zip(*iterable) is its own inverse:

>>> list(source_list) == zip(*zip(*source_list))
True

When unpacking into two lists, this becomes:

>>> list1, list2 = zip(*source_list)
>>> list(source_list) == zip(list1, list2)
True

Addition suggested by rocksportrocker.

How to generate a random alpha-numeric string

import java.util.Random;

public class passGen{
    // Version 1.0
    private static final String dCase = "abcdefghijklmnopqrstuvwxyz";
    private static final String uCase = "ABCDEFGHIJKLMNOPQRSTUVWXYZ";
    private static final String sChar = "!@#$%^&*";
    private static final String intChar = "0123456789";
    private static Random r = new Random();
    private static StringBuilder pass = new StringBuilder();

    public static void main (String[] args) {
        System.out.println ("Generating pass...");
        while (pass.length () != 16){
            int rPick = r.nextInt(4);
            if (rPick == 0){
                int spot = r.nextInt(26);
                pass.append(dCase.charAt(spot));
            } else if (rPick == 1) {
                int spot = r.nextInt(26);
                pass.append(uCase.charAt(spot));
            } else if (rPick == 2) {
                int spot = r.nextInt(8);
                pass.append(sChar.charAt(spot));
            } else {
                int spot = r.nextInt(10);
                pass.append(intChar.charAt(spot));
            }
        }
        System.out.println ("Generated Pass: " + pass.toString());
    }
}

This just adds the password into the string and... yeah, it works well. Check it out... It is very simple; I wrote it.

Calculating the SUM of (Quantity*Price) from 2 different tables

I had the same problem as Marko and come across a solution like this:

/*Create a Table*/
CREATE TABLE tableGrandTotal
(
columnGrandtotal int
)

/*Create a Stored Procedure*/
CREATE PROCEDURE GetGrandTotal
AS

/*Delete the 'tableGrandTotal' table for another usage of the stored procedure*/
DROP TABLE tableGrandTotal

/*Create a new Table which will include just one column*/
CREATE TABLE tableGrandTotal
(
columnGrandtotal int
)

/*Insert the query which returns subtotal for each orderitem row into tableGrandTotal*/
INSERT INTO tableGrandTotal
    SELECT oi.Quantity * p.Price AS columnGrandTotal
        FROM OrderItem oi
        JOIN Product p ON oi.Id = p.Id

/*And return the sum of columnGrandTotal from the newly created table*/    
SELECT SUM(columnGrandTotal) as [Grand Total]
    FROM tableGrandTotal

And just simply use the GetGrandTotal Stored Procedure to retrieve the Grand Total :)

EXEC GetGrandTotal

How to add new line into txt file

No new line:

File.AppendAllText("file.txt", DateTime.Now.ToString());

and then to get a new line after OK:

File.AppendAllText("file.txt", string.Format("{0}{1}", "OK", Environment.NewLine));

uncaught syntaxerror unexpected token U JSON

Most common case of this error happening is using template that is generating the control then changing the way id and/or nameare being generated by 'overriding' default template with something like

@Html.TextBoxFor(m => m, new {Name = ViewData["Name"], id = ViewData["UniqueId"]} )

and then forgetting to change ValidationMessageFor to

@Html.ValidationMessageFor(m => m, null, new { data_valmsg_for = ViewData["Name"] })    

Hope this saves you some time.

URL Encode a string in jQuery for an AJAX request

encodeURIComponent works fine for me. we can give the url like this in ajax call.The code shown below:

  $.ajax({
    cache: false,
    type: "POST",
    url: "http://atandra.mivamerchantdev.com//mm5/json.mvc?Store_Code=ATA&Function=Module&Module_Code=thub_connector&Module_Function=THUB_Request",
    data: "strChannelName=" + $('#txtupdstorename').val() + "&ServiceUrl=" + encodeURIComponent($('#txtupdserviceurl').val()),
    dataType: "HTML",
    success: function (data) {
    },
    error: function (xhr, ajaxOptions, thrownError) {
    }
  });

How do I use cascade delete with SQL Server?

If the one to many relationship is from T1 to T2 then it doesn't represent a function and therefore cannot be used to deduce or infer an inverse function that guarantees the resulting T2 value doesn't omit tuples of T1 join T2 that are deductively valid, because there is no deductively valid inverse function. ( representing functions was the purpose of primary keys. ) The answer in SQL think is yes you can do it. The answer in relational think is no you can't do it. See points of ambiguity in Codd 1970. The relationship would have to be many-to-one from T1 to T2.

How to get the command line args passed to a running process on unix/linux systems?

In addition to all the above ways to convert the text, if you simply use 'strings', it will make the output on separate lines by default. With the added benefit that it may also prevent any chars that may scramble your terminal from appearing.

Both output in one command:

strings /proc//cmdline /proc//environ

The real question is... is there a way to see the real command line of a process in Linux that has been altered so that the cmdline contains the altered text instead of the actual command that was run.

How to iterate through range of Dates in Java?

public static final void generateRange(final Date dateFrom, final Date dateTo)
{
    final Calendar current = Calendar.getInstance();
    current.setTime(dateFrom);

    while (!current.getTime().after(dateTo))
    {
        // TODO

        current.add(Calendar.DATE, 1);
    }
}

MySql difference between two timestamps in days?

SELECT DATEDIFF(max_date, min_date) as days from my table. This works even if the col max_date and min_date are in string data types.

Difference between "and" and && in Ruby?

and is the same as && but with lower precedence. They both use short-circuit evaluation.

WARNING: and even has lower precedence than = so you'll usually want to avoid and. An example when and should be used can be found in the Rails Guide under "Avoiding Double Render Errors".

Pad a string with leading zeros so it's 3 characters long in SQL Server 2008

I came here specifically to work out how I could convert my timezoneoffset to a timezone string for converting dates to DATETIMEOFFSET in SQL Server 2008. Gross, but necessary.

So I need 1 method that will cope with negative and positive numbers, formatting them to two characters with a leading zero if needed. Anons answer got me close, but negative timezone values would come out as 0-5 rather than the required -05

So with a bit of a tweak on his answer, this works for all timezone hour conversions

DECLARE @n INT = 13 -- Works with -13, -5, 0, 5, etc
SELECT CASE 
    WHEN @n < 0 THEN '-' + REPLACE(STR(@n * -1 ,2),' ','0') 
    ELSE '+' + REPLACE(STR(@n,2),' ','0') END + ':00'

Test for array of string type in TypeScript

there is a little problem here because the

if (typeof item !== 'string') {
    return false
}

will not stop the foreach. So the function will return true even if the array does contain none string values.

This seems to wok for me:

function isStringArray(value: any): value is number[] {
  if (Object.prototype.toString.call(value) === '[object Array]') {
     if (value.length < 1) {
       return false;
     } else {
       return value.every((d: any) => typeof d === 'string');
     }
  }
  return false;
}

Greetings, Hans

How do I prevent and/or handle a StackOverflowException?

NOTE The question in the bounty by @WilliamJockusch and the original question are different.

This answer is about StackOverflow's in the general case of third-party libraries and what you can/can't do with them. If you're looking about the special case with XslTransform, see the accepted answer.


Stack overflows happen because the data on the stack exceeds a certain limit (in bytes). The details of how this detection works can be found here.

I'm wondering if there is a general way to track down StackOverflowExceptions. In other words, suppose I have infinite recursion somewhere in my code, but I have no idea where. I want to track it down by some means that is easier than stepping through code all over the place until I see it happening. I don't care how hackish it is.

As I mentioned in the link, detecting a stack overflow from static code analysis would require solving the halting problem which is undecidable. Now that we've established that there is no silver bullet, I can show you a few tricks that I think helps track down the problem.

I think this question can be interpreted in different ways, and since I'm a bit bored :-), I'll break it down into different variations.

Detecting a stack overflow in a test environment

Basically the problem here is that you have a (limited) test environment and want to detect a stack overflow in an (expanded) production environment.

Instead of detecting the SO itself, I solve this by exploiting the fact that the stack depth can be set. The debugger will give you all the information you need. Most languages allow you to specify the stack size or the max recursion depth.

Basically I try to force a SO by making the stack depth as small as possible. If it doesn't overflow, I can always make it bigger (=in this case: safer) for the production environment. The moment you get a stack overflow, you can manually decide if it's a 'valid' one or not.

To do this, pass the stack size (in our case: a small value) to a Thread parameter, and see what happens. The default stack size in .NET is 1 MB, we're going to use a way smaller value:

class StackOverflowDetector
{
    static int Recur()
    {
        int variable = 1;
        return variable + Recur();
    }

    static void Start()
    {
        int depth = 1 + Recur();
    }

    static void Main(string[] args)
    {
        Thread t = new Thread(Start, 1);
        t.Start();
        t.Join();
        Console.WriteLine();
        Console.ReadLine();
    }
}

Note: we're going to use this code below as well.

Once it overflows, you can set it to a bigger value until you get a SO that makes sense.

Creating exceptions before you SO

The StackOverflowException is not catchable. This means there's not much you can do when it has happened. So, if you believe something is bound to go wrong in your code, you can make your own exception in some cases. The only thing you need for this is the current stack depth; there's no need for a counter, you can use the real values from .NET:

class StackOverflowDetector
{
    static void CheckStackDepth()
    {
        if (new StackTrace().FrameCount > 10) // some arbitrary limit
        {
            throw new StackOverflowException("Bad thread.");
        }
    }

    static int Recur()
    {
        CheckStackDepth();
        int variable = 1;
        return variable + Recur();
    }

    static void Main(string[] args)
    {
        try
        {
            int depth = 1 + Recur();
        }
        catch (ThreadAbortException e)
        {
            Console.WriteLine("We've been a {0}", e.ExceptionState);
        }
        Console.WriteLine();
        Console.ReadLine();
    }
}

Note that this approach also works if you are dealing with third-party components that use a callback mechanism. The only thing required is that you can intercept some calls in the stack trace.

Detection in a separate thread

You explicitly suggested this, so here goes this one.

You can try detecting a SO in a separate thread.. but it probably won't do you any good. A stack overflow can happen fast, even before you get a context switch. This means that this mechanism isn't reliable at all... I wouldn't recommend actually using it. It was fun to build though, so here's the code :-)

class StackOverflowDetector
{
    static int Recur()
    {
        Thread.Sleep(1); // simulate that we're actually doing something :-)
        int variable = 1;
        return variable + Recur();
    }

    static void Start()
    {
        try
        {
            int depth = 1 + Recur();
        }
        catch (ThreadAbortException e)
        {
            Console.WriteLine("We've been a {0}", e.ExceptionState);
        }
    }

    static void Main(string[] args)
    {
        // Prepare the execution thread
        Thread t = new Thread(Start);
        t.Priority = ThreadPriority.Lowest;

        // Create the watch thread
        Thread watcher = new Thread(Watcher);
        watcher.Priority = ThreadPriority.Highest;
        watcher.Start(t);

        // Start the execution thread
        t.Start();
        t.Join();

        watcher.Abort();
        Console.WriteLine();
        Console.ReadLine();
    }

    private static void Watcher(object o)
    {
        Thread towatch = (Thread)o;

        while (true)
        {
            if (towatch.ThreadState == System.Threading.ThreadState.Running)
            {
                towatch.Suspend();
                var frames = new System.Diagnostics.StackTrace(towatch, false);
                if (frames.FrameCount > 20)
                {
                    towatch.Resume();
                    towatch.Abort("Bad bad thread!");
                }
                else
                {
                    towatch.Resume();
                }
            }
        }
    }
}

Run this in the debugger and have fun of what happens.

Using the characteristics of a stack overflow

Another interpretation of your question is: "Where are the pieces of code that could potentially cause a stack overflow exception?". Obviously the answer of this is: all code with recursion. For each piece of code, you can then do some manual analysis.

It's also possible to determine this using static code analysis. What you need to do for that is to decompile all methods and figure out if they contain an infinite recursion. Here's some code that does that for you:

// A simple decompiler that extracts all method tokens (that is: call, callvirt, newobj in IL)
internal class Decompiler
{
    private Decompiler() { }

    static Decompiler()
    {
        singleByteOpcodes = new OpCode[0x100];
        multiByteOpcodes = new OpCode[0x100];
        FieldInfo[] infoArray1 = typeof(OpCodes).GetFields();
        for (int num1 = 0; num1 < infoArray1.Length; num1++)
        {
            FieldInfo info1 = infoArray1[num1];
            if (info1.FieldType == typeof(OpCode))
            {
                OpCode code1 = (OpCode)info1.GetValue(null);
                ushort num2 = (ushort)code1.Value;
                if (num2 < 0x100)
                {
                    singleByteOpcodes[(int)num2] = code1;
                }
                else
                {
                    if ((num2 & 0xff00) != 0xfe00)
                    {
                        throw new Exception("Invalid opcode: " + num2.ToString());
                    }
                    multiByteOpcodes[num2 & 0xff] = code1;
                }
            }
        }
    }

    private static OpCode[] singleByteOpcodes;
    private static OpCode[] multiByteOpcodes;

    public static MethodBase[] Decompile(MethodBase mi, byte[] ildata)
    {
        HashSet<MethodBase> result = new HashSet<MethodBase>();

        Module module = mi.Module;

        int position = 0;
        while (position < ildata.Length)
        {
            OpCode code = OpCodes.Nop;

            ushort b = ildata[position++];
            if (b != 0xfe)
            {
                code = singleByteOpcodes[b];
            }
            else
            {
                b = ildata[position++];
                code = multiByteOpcodes[b];
                b |= (ushort)(0xfe00);
            }

            switch (code.OperandType)
            {
                case OperandType.InlineNone:
                    break;
                case OperandType.ShortInlineBrTarget:
                case OperandType.ShortInlineI:
                case OperandType.ShortInlineVar:
                    position += 1;
                    break;
                case OperandType.InlineVar:
                    position += 2;
                    break;
                case OperandType.InlineBrTarget:
                case OperandType.InlineField:
                case OperandType.InlineI:
                case OperandType.InlineSig:
                case OperandType.InlineString:
                case OperandType.InlineTok:
                case OperandType.InlineType:
                case OperandType.ShortInlineR:
                    position += 4;
                    break;
                case OperandType.InlineR:
                case OperandType.InlineI8:
                    position += 8;
                    break;
                case OperandType.InlineSwitch:
                    int count = BitConverter.ToInt32(ildata, position);
                    position += count * 4 + 4;
                    break;

                case OperandType.InlineMethod:
                    int methodId = BitConverter.ToInt32(ildata, position);
                    position += 4;
                    try
                    {
                        if (mi is ConstructorInfo)
                        {
                            result.Add((MethodBase)module.ResolveMember(methodId, mi.DeclaringType.GetGenericArguments(), Type.EmptyTypes));
                        }
                        else
                        {
                            result.Add((MethodBase)module.ResolveMember(methodId, mi.DeclaringType.GetGenericArguments(), mi.GetGenericArguments()));
                        }
                    }
                    catch { } 
                    break;


                default:
                    throw new Exception("Unknown instruction operand; cannot continue. Operand type: " + code.OperandType);
            }
        }
        return result.ToArray();
    }
}

class StackOverflowDetector
{
    // This method will be found:
    static int Recur()
    {
        CheckStackDepth();
        int variable = 1;
        return variable + Recur();
    }

    static void Main(string[] args)
    {
        RecursionDetector();
        Console.WriteLine();
        Console.ReadLine();
    }

    static void RecursionDetector()
    {
        // First decompile all methods in the assembly:
        Dictionary<MethodBase, MethodBase[]> calling = new Dictionary<MethodBase, MethodBase[]>();
        var assembly = typeof(StackOverflowDetector).Assembly;

        foreach (var type in assembly.GetTypes())
        {
            foreach (var member in type.GetMembers(BindingFlags.Public | BindingFlags.NonPublic | BindingFlags.Static | BindingFlags.Instance).OfType<MethodBase>())
            {
                var body = member.GetMethodBody();
                if (body!=null)
                {
                    var bytes = body.GetILAsByteArray();
                    if (bytes != null)
                    {
                        // Store all the calls of this method:
                        var calls = Decompiler.Decompile(member, bytes);
                        calling[member] = calls;
                    }
                }
            }
        }

        // Check every method:
        foreach (var method in calling.Keys)
        {
            // If method A -> ... -> method A, we have a possible infinite recursion
            CheckRecursion(method, calling, new HashSet<MethodBase>());
        }
    }

Now, the fact that a method cycle contains recursion, is by no means a guarantee that a stack overflow will happen - it's just the most likely precondition for your stack overflow exception. In short, this means that this code will determine the pieces of code where a stack overflow can occur, which should narrow down most code considerably.

Yet other approaches

There are some other approaches you can try that I haven't described here.

  1. Handling the stack overflow by hosting the CLR process and handling it. Note that you still cannot 'catch' it.
  2. Changing all IL code, building another DLL, adding checks on recursion. Yes, that's quite possible (I've implemented it in the past :-); it's just difficult and involves a lot of code to get it right.
  3. Use the .NET profiling API to capture all method calls and use that to figure out stack overflows. For example, you can implement checks that if you encounter the same method X times in your call tree, you give a signal. There's a project here that will give you a head start.

Using jQuery's ajax method to retrieve images as a blob

A big thank you to @Musa and here is a neat function that converts the data to a base64 string. This may come handy to you when handling a binary file (pdf, png, jpeg, docx, ...) file in a WebView that gets the binary file but you need to transfer the file's data safely into your app.

// runs a get/post on url with post variables, where:
// url ... your url
// post ... {'key1':'value1', 'key2':'value2', ...}
//          set to null if you need a GET instead of POST req
// done ... function(t) called when request returns
function getFile(url, post, done)
{
   var postEnc, method;
   if (post == null)
   {
      postEnc = '';
      method = 'GET';
   }
   else
   {
      method = 'POST';
      postEnc = new FormData();
      for(var i in post)
         postEnc.append(i, post[i]);
   }
   var xhr = new XMLHttpRequest();
   xhr.onreadystatechange = function() {
      if (this.readyState == 4 && this.status == 200)
      {
         var res = this.response;
         var reader = new window.FileReader();
         reader.readAsDataURL(res); 
         reader.onloadend = function() { done(reader.result.split('base64,')[1]); }
      }
   }
   xhr.open(method, url);
   xhr.setRequestHeader('Content-type', 'application/x-www-form-urlencoded');
   xhr.send('fname=Henry&lname=Ford');
   xhr.responseType = 'blob';
   xhr.send(postEnc);
}

What arguments are passed into AsyncTask<arg1, arg2, arg3>?

I'm too late to the party but thought this might help someone.

java.sql.SQLException: Fail to convert to internal representation

Check your Entity class. Use String instead of Long and float instead of double .

How do I check when a UITextField changes?

txf_Subject.addTarget(self, action:#selector(didChangeFirstText), for: .editingChanged)

@objc func didChangeText(textField:UITextField) {
    let str = textField.text
    if(str?.contains(" "))!{
        let newstr = str?.replacingOccurrences(of: " ", with: "")
        textField.text = newstr
    }
}

@objc func didChangeFirstText(textField:UITextField) {
    if(textField.text == " "){
        textField.text = ""
    }
}

How to start up spring-boot application via command line?

Run Spring Boot app using Maven

You can also use Maven plugin to run your Spring Boot app. Use the below example to run your Spring Boot app with Maven plugin:

mvn spring-boot:run

Run Spring Boot App with Gradle

And if you use Gradle you can run the Spring Boot app with the following command:

gradle bootRun

REST vs JSON-RPC?

It would be better to choose JSON-RPC between REST and JSON-RPC to develop an API for a web application that is easier to understand. JSON-RPC is preferred because its mapping to method calls and communications can be easily understood.

Choosing the most suitable approach depends on the constraints or principal objective. For example, as far as performance is a major trait, it is advisable to go for JSON-RPC (for example, High Performance Computing). However, if the principal objective is to be agnostic in order to offer a generic interface to be inferred by others, it is advisable to go for REST. If you both goals are needed to be achieved, it is advisable to include both protocols.

The fact which actually splits REST from JSON-RPC is that it trails a series of carefully thought out constraints- confirming architectural flexibility. The constraints take in ensuring that the client as well as server are able to grow independently of each other (changes can be made without messing up with the application of client), the calls are stateless (the state is regarded as hypermedia), a uniform interface is offered for interactions, the API is advanced on a layered system (Hall, 2010). JSON-RPC is rapid and easy to consume, however as mentioned resources as well as parameters are tightly coupled and it is likely to depend on verbs (api/addUser, api/deleteUser) using GET/ POST whereas REST delivers loosely coupled resources (api/users) in a HTTP. REST API depends up on several HTTP methods such as GET, PUT, POST, DELETE, PATCH. REST is slightly tougher for inexperienced developers to implement.

JSON (denoted as JavaScript Object Notation) being a lightweight data-interchange format, is easy for humans to read as well as write. It is hassle free for machines to parse and generate. JSON is a text format which is entirely language independent but practices conventions that are acquainted to programmers of the family of languages, consisting of C#, C, C++, Java, Perl, JavaScript, Python, and numerous others. Such properties make JSON a perfect data-interchange language and a better choice to opt for.

How to compare dates in datetime fields in Postgresql?

@Nicolai is correct about casting and why the condition is false for any data. i guess you prefer the first form because you want to avoid date manipulation on the input string, correct? you don't need to be afraid:

SELECT *
FROM table
WHERE update_date >= '2013-05-03'::date
AND update_date < ('2013-05-03'::date + '1 day'::interval);

Adding horizontal spacing between divs in Bootstrap 3

Do you mean something like this? JSFiddle

Attribute used:

margin-left: 50px;

How to execute a query in ms-access in VBA code?

How about something like this...

Dim rs As RecordSet
Set rs = Currentdb.OpenRecordSet("SELECT PictureLocation, ID FROM MyAccessTable;")

Do While Not rs.EOF
   Debug.Print rs("PictureLocation") & " - " & rs("ID")
   rs.MoveNext
Loop

How to custom switch button?

you can use the following code to change color and text :

<org.jraf.android.backport.switchwidget.Switch
                        android:id="@+id/th"
                        android:layout_width="match_parent"
                        android:layout_height="wrap_content"
                        app:thumb="@drawable/apptheme_switch_inner_holo_light"
                        app:track="@drawable/apptheme_switch_track_holo_light"
                        app:textOn="@string/switch_yes"
                        app:textOff="@string/switch_no"
                        android:textColor="#000000"
                        />

Create a xml named colors.xml in res/values folder:

<?xml version="1.0" encoding="utf-8"?>
<resources>
    <color name="red">#ff0000</color>
    <color name="green">#00ff00</color>
</resources>

In drawable folder, create a xml file my_btn_toggle.xml:

  <selector xmlns:android="http://schemas.android.com/apk/res/android">
        <item android:state_checked="false" android:drawable="@color/red"  />
        <item android:state_checked="true" android:drawable="@color/green"  />
    </selector>

and in xml section defining your toggle button add:

android:background="@drawable/my_btn_toggle

to change the color of textOn and textOffuse

android:switchTextAppearance="@style/Switch"

Jquery Value match Regex

Change it to this:

var email = /^[A-Z0-9._%+-]+@[A-Z0-9.-]+\.[A-Z]{2,4}$/i;

This is a regular expression literal that is passed the i flag which means to be case insensitive.

Keep in mind that email address validation is hard (there is a 4 or 5 page regular expression at the end of Mastering Regular Expressions demonstrating this) and your expression certainly will not capture all valid e-mail addresses.

C# switch statement limitations - why?

Mostly, those restrictions are in place because of language designers. The underlying justification may be compatibility with languange history, ideals, or simplification of compiler design.

The compiler may (and does) choose to:

  • create a big if-else statement
  • use a MSIL switch instruction (jump table)
  • build a Generic.Dictionary<string,int32>, populate it on first use, and call Generic.Dictionary<>::TryGetValue() for a index to pass to a MSIL switch instruction (jump table)
  • use a combination of if-elses & MSIL "switch" jumps

The switch statement IS NOT a constant time branch. The compiler may find short-cuts (using hash buckets, etc), but more complicated cases will generate more complicated MSIL code with some cases branching out earlier than others.

To handle the String case, the compiler will end up (at some point) using a.Equals(b) (and possibly a.GetHashCode() ). I think it would be trival for the compiler to use any object that satisfies these constraints.

As for the need for static case expressions... some of those optimisations (hashing, caching, etc) would not be available if the case expressions weren't deterministic. But we've already seen that sometimes the compiler just picks the simplistic if-else-if-else road anyway...

Edit: lomaxx - Your understanding of the "typeof" operator is not correct. The "typeof" operator is used to obtain the System.Type object for a type (nothing to do with its supertypes or interfaces). Checking run-time compatibility of an object with a given type is the "is" operator's job. The use of "typeof" here to express an object is irrelevant.

What is IllegalStateException?

public class UserNotFoundException extends Exception {
    public UserNotFoundException(String message) {
        super(message)

Non-resolvable parent POM using Maven 3.0.3 and relativePath notation

'parent.relativePath' points at wrong local POM @ myGroup:myParentArtifactId:1.0, C:\myProjectDir\parent\pom.xml

This indicates that maven did search locally for the parent pom, but found that it was not the correct pom.

  • Does pom.xml of parentpom correctly define the parent pom as the pom.xml of rootpom?
  • Does rootpom folder contain pom.xml as well as the paretpom folder?

Spring Data: "delete by" is supported?

Derivation of delete queries using given method name is supported starting with version 1.6.0.RC1 of Spring Data JPA. The keywords remove and delete are supported. As return value one can choose between the number or a list of removed entities.

Long removeByLastname(String lastname);

List<User> deleteByLastname(String lastname);

How do I line up 3 divs on the same row?

2019 answer:

Using CSS grid:

.parent {
  display: grid;
  grid-template-columns: 1fr 1fr 1fr;
  grid-template-rows: 1fr;
}

month name to month number and vice versa in python

To get the full calendar name from the month number, you can use calendar.month_name. Please see the documentation for more details: https://docs.python.org/2/library/calendar.html

month_no = 1
month = calendar.month_name[month_no]

# month provides "January":
print(month)


SQLAlchemy create_all() does not create tables

This is probably not the main reason why the create_all() method call doesn't work for people, but for me, the cobbled together instructions from various tutorials have it such that I was creating my db in a request context, meaning I have something like:

# lib/db.py
from flask import g, current_app
from flask_sqlalchemy import SQLAlchemy

def get_db():
  if 'db' not in g:
    g.db = SQLAlchemy(current_app)
  return g.db

I also have a separate cli command that also does the create_all:

# tasks/db.py
from lib.db import get_db

@current_app.cli.command('init-db')
def init_db():
  db = get_db()
  db.create_all()

I also am using a application factory.

When the cli command is run, a new app context is used, which means a new db is used. Furthermore, in this world, an import model in the init_db method does not do anything, because it may be that your model file was already loaded(and associated with a separate db).

The fix that I came around to was to make sure that the db was a single global reference:

# lib/db.py
from flask import g, current_app
from flask_sqlalchemy import SQLAlchemy

db = None
def get_db():
  global db
  if not db:
    db = SQLAlchemy(current_app)
  return db

I have not dug deep enough into flask, sqlalchemy, or flask-sqlalchemy to understand if this means that requests to the db from multiple threads are safe, but if you're reading this you're likely stuck in the baby stages of understanding these concepts too.

How to add one day to a date?

U can try java.util.Date library like this way-

int no_of_day_to_add = 1;

Date today = new Date();
Date tomorrow = new Date( today.getYear(), today.getMonth(), today.getDate() + no_of_day_to_add );

Change value of no_of_day_to_add as you want.

I have set value of no_of_day_to_add to 1 because u wanted only one day to add.

More can be found in this documentation.

.htaccess File Options -Indexes on Subdirectories

htaccess files affect the directory they are placed in and all sub-directories, that is an htaccess file located in your root directory (yoursite.com) would affect yoursite.com/content, yoursite.com/content/contents, etc.

http://www.javascriptkit.com/howto/htaccess.shtml

SQL grouping by month and year

Yet another alternative: 

Select FORMAT(date,'MM.yy')
...
...
group by FORMAT(date,'MM.yy')

Decrementing for loops

You need to give the range a -1 step

 for i in range(10,0,-1):
    print i

Return from lambda forEach() in java

You can also throw an exception:

Note:

For the sake of readability each step of stream should be listed in new line.

players.stream()
       .filter(player -> player.getName().contains(name))
       .findFirst()
       .orElseThrow(MyCustomRuntimeException::new);

if your logic is loosely "exception driven" such as there is one place in your code that catches all exceptions and decides what to do next. Only use exception driven development when you can avoid littering your code base with multiples try-catch and throwing these exceptions are for very special cases that you expect them and can be handled properly.)

Check if the number is integer

Another alternative is to check the fractional part:

x%%1==0

or, if you want to check within a certain tolerance:

min(abs(c(x%%1, x%%1-1))) < tol

How to render string with html tags in Angular 4+?

Use one way flow syntax property binding:

<div [innerHTML]="comment"></div>

From angular docs: "Angular recognizes the value as unsafe and automatically sanitizes it, which removes the <script> tag but keeps safe content such as the <b> element."

How do I run a Python script on my web server?

Very simply, you can rename your Python script to "pythonscript.cgi". Post that in your cgi-bin directory, add the appropriate permissions and browse to it.

This is a great link you can start with.

Here's another good one.

Hope that helps.


EDIT (09/12/2015): The second link has long been removed. Replaced it with one that provides information referenced from the original.

Inline SVG in CSS

I found one solution for SVG. But it is work only for Webkit, I just want share my workaround with you. In my example is shown how to use SVG element from DOM as background through a filter (background-image: url('#glyph') is not working).

Features needed for this SVG icon render:

  1. Applying SVG filter effects to HTML elements using CSS (IE and Edge not supports)
  2. feImage fragment load supporting (firefox not supports)

_x000D_
_x000D_
.test {_x000D_
  /*  background-image: url('#glyph');_x000D_
    background-size:100% 100%;*/_x000D_
    filter: url(#image); _x000D_
    height:100px;_x000D_
    width:100px;_x000D_
}_x000D_
.test:before {_x000D_
   display:block;_x000D_
   content:'';_x000D_
   color:transparent;_x000D_
}_x000D_
.test2{_x000D_
  width:100px;_x000D_
  height:100px;_x000D_
}_x000D_
.test2:before {_x000D_
   display:block;_x000D_
   content:'';_x000D_
   color:transparent;_x000D_
   filter: url(#image); _x000D_
   height:100px;_x000D_
   width:100px;_x000D_
}
_x000D_
<svg style="height:0;width:0;" version="1.1" viewbox="0 0 100 100"_x000D_
     xmlns="http://www.w3.org/2000/svg"_x000D_
     xmlns:xlink="http://www.w3.org/1999/xlink">_x000D_
 <defs>_x000D_
     <g id="glyph">_x000D_
          <path id="heart" d="M100 34.976c0 8.434-3.635 16.019-9.423 21.274h0.048l-31.25 31.25c-3.125 3.125-6.25 6.25-9.375 6.25s-6.25-3.125-9.375-6.25l-31.202-31.25c-5.788-5.255-9.423-12.84-9.423-21.274 0-15.865 12.861-28.726 28.726-28.726 8.434 0 16.019 3.635 21.274 9.423 5.255-5.788 12.84-9.423 21.274-9.423 15.865 0 28.726 12.861 28.726 28.726z" fill="crimson"/>_x000D_
     </g>_x000D_
    <svg id="resized-glyph"  x="0%" y="0%" width="24" height="24" viewBox="0 0 100 100" class="icon shape-codepen">_x000D_
      <use xlink:href="#glyph"></use>_x000D_
    </svg>_x000D_
     <filter id="image">_x000D_
       <feImage xlink:href="#resized-glyph" x="0%" y="0%" width="100%" height="100%" result="res"/>_x000D_
       <feComposite operator="over" in="res" in2="SourceGraphic"/>_x000D_
    </filter>_x000D_
 </defs>_x000D_
</svg>_x000D_
<div class="test">_x000D_
</div>_x000D_
<div class="test2">_x000D_
</div>
_x000D_
_x000D_
_x000D_

One more solution, is use url encode

_x000D_
_x000D_
var container = document.querySelector(".container");_x000D_
var svg = document.querySelector("svg");_x000D_
var svgText = (new XMLSerializer()).serializeToString(svg);_x000D_
container.style.backgroundImage = `url(data:image/svg+xml;utf8,${encodeURIComponent(svgText)})`;
_x000D_
.container{_x000D_
  height:50px;_x000D_
  width:250px;_x000D_
  display:block;_x000D_
  background-position: center center;_x000D_
  background-repeat: no-repeat;_x000D_
  background-size: contain;_x000D_
}
_x000D_
<svg  height="100" width="500" xmlns="http://www.w3.org/2000/svg">_x000D_
    <ellipse cx="240" cy="50" rx="220" ry="30" style="fill:yellow" />_x000D_
</svg>_x000D_
<div class="container"></div>
_x000D_
_x000D_
_x000D_

How do I sort a dictionary by value?

Because of requirements to retain backward compatability with older versions of Python I think the OrderedDict solution is very unwise. You want something that works with Python 2.7 and older versions.

But the collections solution mentioned in another answer is absolutely superb, because you retrain a connection between the key and value which in the case of dictionaries is extremely important.

I don't agree with the number one choice presented in another answer, because it throws away the keys.

I used the solution mentioned above (code shown below) and retained access to both keys and values and in my case the ordering was on the values, but the importance was the ordering of the keys after ordering the values.

from collections import Counter

x = {'hello':1, 'python':5, 'world':3}
c=Counter(x)
print c.most_common()


>> [('python', 5), ('world', 3), ('hello', 1)]

How to add an existing folder with files to SVN?

I don't use commands. You should be able to do this using the GUI:

  • Right-click an empty space in your My Documents folder, select TortoiseSVN > Repo-browser.
  • Enter http://subversion... (your URL path to your Subversion server/directory you will save to) as your path and select OK
  • Right-click the root directory in Repo and select Add folder. Give it the name of your project and create it.
  • Right-click the project folder in the Repo-browser and select Checkout. The Checkout directory will be your Visual Studio\Projects\{your project} folder. Select OK.
  • You will receive a warning that the folder is not empty. Say Yes to checkout/export to that folder - it will not overwrite your project files.
  • Open your project folder. You will see question marks on folders that are associated with your VS project that have not yet been added to Subversion. Select those folders using Ctrl + Click, then right-click one of the selected items and select TortoiseSVN > Add
  • Select OK on the prompt
  • Your files should add. Select OK on the Add Finished! dialog
  • Right-click in an empty area of the folder and select Refresh. You’ll see “+” icons on the folders/files, now
  • Right-click an empty area in the folder once again and select SVN Commit
  • Add a message regarding what you are committing and click OK

java.lang.IllegalStateException: Fragment not attached to Activity

So the base idea is that you are running a UI operation on a fragment that is getting in the onDetach lifecycle.

When this is happening the fragment is getting off the stack and losing the context of the Activity.

So when you call UI related functions for example calling the progress spinner and you want to leave the fragment check if the Fragment is added to the stack, like this:

if(isAdded){ progressBar.visibility=View.VISIBLE }

PHP Function Comments

You can get the comments of a particular method by using the ReflectionMethod class and calling ->getDocComment().

http://www.php.net/manual/en/reflectionclass.getdoccomment.php

Solution for "Fatal error: Maximum function nesting level of '100' reached, aborting!" in PHP

You could convert your recursive code into an iterative code, which simulates the recursion. This means that you have to push the current status (url, document, position in document etc.) into an array, when you reach a link, and pop it out of the array, when this link has finished.

When to use HashMap over LinkedList or ArrayList and vice-versa

Lists and Maps are different data structures. Maps are used for when you want to associate a key with a value and Lists are an ordered collection.

Map is an interface in the Java Collection Framework and a HashMap is one implementation of the Map interface. HashMap are efficient for locating a value based on a key and inserting and deleting values based on a key. The entries of a HashMap are not ordered.

ArrayList and LinkedList are an implementation of the List interface. LinkedList provides sequential access and is generally more efficient at inserting and deleting elements in the list, however, it is it less efficient at accessing elements in a list. ArrayList provides random access and is more efficient at accessing elements but is generally slower at inserting and deleting elements.

Using ZXing to create an Android barcode scanning app

Barcode Detection is now available in Google Play services. Code lab of the setup process, here are the api docs, and a sample project.

SQL Add foreign key to existing column

Error indicates that there is no UserID column in your Employees table. Try adding the column first and then re-run the statement.

ALTER TABLE Employees
ADD CONSTRAINT FK_ActiveDirectories_UserID FOREIGN KEY (UserID)
    REFERENCES ActiveDirectories(id);

HTTP Error 403.14 - Forbidden - The Web server is configured to not list the contents of this directory

There can be multiple reasons for the issue. One that worked for me on IIS 8.5 was as follow

Steps

  1. Type "turn windows features on or off" in search.
  2. Click on "Add Roles and features" in Server Manager.
  3. In Wizard scroll down to the Web server and select : Web Server -> Application Development. Select all except CGI from the list as shown in the screen shot

enter image description here

  1. Finally hit next and Install.
  2. Restart IIS

Your website may start working.

Git Symlinks in Windows

One simple trick we use is to just call git add --all twice in a row.

For example, our Windows 7 commit script calls:

$ git add --all
$ git add --all

The first add treats the link as text and adds the folders for delete.

The second add traverses the link correctly and undoes the delete by restoring the files.

It's less elegant than some of the other proposed solutions but it is a simple fix to some of our legacy environments that got symlinks added.

Rails: How to reference images in CSS within Rails 4

None of the answers says about the way, when I'll have .css.erb extension, how to reference images. For me worked both in production and development as well :

2.3.1 CSS and ERB

The asset pipeline automatically evaluates ERB. This means if you add an erb extension to a CSS asset (for example, application.css.erb), then helpers like asset_path are available in your CSS rules:

.class { background-image: url(<%= asset_path 'image.png' %>) }

This writes the path to the particular asset being referenced. In this example, it would make sense to have an image in one of the asset load paths, such as app/assets/images/image.png, which would be referenced here. If this image is already available in public/assets as a fingerprinted file, then that path is referenced.

If you want to use a data URI - a method of embedding the image data directly into the CSS file - you can use the asset_data_uri helper.

.logo { background: url(<%= asset_data_uri 'logo.png' %>) }

This inserts a correctly-formatted data URI into the CSS source.

Note that the closing tag cannot be of the style -%>.

How to check if variable is array?... or something array-like

<?php
$var = new ArrayIterator();

var_dump(is_array($var), ($var instanceof ArrayIterator));

returns bool(false) or bool(true)

Drop Down Menu/Text Field in one

You can do this natively with HTML5 <datalist>:

_x000D_
_x000D_
<label>Choose a browser from this list:_x000D_
<input list="browsers" name="myBrowser" /></label>_x000D_
<datalist id="browsers">_x000D_
  <option value="Chrome">_x000D_
  <option value="Firefox">_x000D_
  <option value="Internet Explorer">_x000D_
  <option value="Opera">_x000D_
  <option value="Safari">_x000D_
  <option value="Microsoft Edge">_x000D_
</datalist>
_x000D_
_x000D_
_x000D_

What is the difference between Cloud Computing and Grid Computing?

Grid computing is where more than one computer coordinates to solve a problem together. Often used for problems involving a lot of number crunching, which can be easily parallelisable.

Cloud computing is where an application doesn't access resources it requires directly, rather it accesses them through something like a service. So instead of talking to a specific hard drive for storage, and a specific CPU for computation, etc. it talks to some service that provides these resources. The service then maps any requests for resources to its physical resources, in order to provide for the application. Usually the service has access to a large amount of physical resources, and can dynamically allocate them as they are needed.

In this way, if an application requires only a small amount of some resource, say computation, then the service only allocates a small amount, say on a single physical CPU (that may be shared with some other application using the service). If the application requires a large amount of some resource, then the service allocates that large amount, say a grid of CPUs. The application is relatively oblivious to this, and all the complex handling and coordination is performed by the service, not the application. In this way the application can scale well.

For example a web site written "on the cloud" may share a server with many other web sites while it has a low amount of traffic, but may be moved to its own dedicated server, or grid of servers, if it ever has massive amounts of traffic. This is all handled by the cloud service, so the application shouldn't have to be modified drastically to cope.

A cloud would usually use a grid. A grid is not necessarily a cloud or part of a cloud.

Wikipedia articles: Grid computing, Cloud computing.

Convert seconds to Hour:Minute:Second

The following codes can display total hours plus minutes and seconds accurately

$duration_in_seconds = 86401;
if($duration_in_seconds>0)
{
    echo floor($duration_in_seconds/3600).gmdate(":i:s", $duration_in_seconds%3600);
}
else
{
    echo "00:00:00";
}

Ignore 'Security Warning' running script from command line

For those who want to access a file from an already loaded PowerShell session, either use Unblock-File to mark the file as safe (though you already need to have set a relaxed execution policy like Unrestricted for this to work), or change the execution policy just for the current PowerShell session:

Set-ExecutionPolicy -ExecutionPolicy Bypass -Scope Process

Getting XML Node text value with Java DOM

If your XML goes quite deep, you might want to consider using XPath, which comes with your JRE, so you can access the contents far more easily using:

String text = xp.evaluate("//add[@job='351']/tag[position()=1]/text()", 
    document.getDocumentElement());

Full example:

import static org.junit.Assert.assertEquals;
import java.io.StringReader;    
import javax.xml.parsers.DocumentBuilder;
import javax.xml.parsers.DocumentBuilderFactory;
import javax.xml.xpath.XPath;
import javax.xml.xpath.XPathFactory;    
import org.junit.Before;
import org.junit.Test;
import org.w3c.dom.Document;
import org.xml.sax.InputSource;

public class XPathTest {

    private Document document;

    @Before
    public void setup() throws Exception {
        String xml = "<add job=\"351\"><tag>foobar</tag><tag>foobar2</tag></add>";
        DocumentBuilderFactory dbf = DocumentBuilderFactory.newInstance();
        DocumentBuilder db = dbf.newDocumentBuilder();
        document = db.parse(new InputSource(new StringReader(xml)));
    }

    @Test
    public void testXPath() throws Exception {
        XPathFactory xpf = XPathFactory.newInstance();
        XPath xp = xpf.newXPath();
        String text = xp.evaluate("//add[@job='351']/tag[position()=1]/text()",
                document.getDocumentElement());
        assertEquals("foobar", text);
    }
}

How to format number of decimal places in wpf using style/template?

The accepted answer does not show 0 in integer place on giving input like 0.299. It shows .3 in WPF UI. So my suggestion to use following string format

<TextBox Text="{Binding Value,  StringFormat={}{0:#,0.0}}" 

Cannot inline bytecode built with JVM target 1.8 into bytecode that is being built with JVM target 1.6

Setting sourceCompatibility = JavaVersion.VERSION_1_8 enables desugaring, but it is currently unable to desugar all the Java 8 features that the Kotlin compiler uses.

enter image description here

Fix - Setting kotlinOptions.jvmTarget to JavaVersion.VERSION_1_8 in the app module Gradle would fix the issue.

Use Java 8 language features: https://developer.android.com/studio/write/java8-support

android {
  ...
  // Configure only for each module that uses Java 8
  // language features (either in its source code or
  // through dependencies).
  compileOptions {
    sourceCompatibility JavaVersion.VERSION_1_8
    targetCompatibility JavaVersion.VERSION_1_8
  }
  // For Kotlin projects
  kotlinOptions {
    jvmTarget = "1.8"
  }
}

TypeError: only integer scalar arrays can be converted to a scalar index with 1D numpy indices array

Another case that could cause this error is

>>> np.ndindex(np.random.rand(60,60))
TypeError: only integer scalar arrays can be converted to a scalar index

Using the actual shape will fix it.

>>> np.ndindex(np.random.rand(60,60).shape)
<numpy.ndindex object at 0x000001B887A98880>

How to compare two colors for similarity/difference

A color value has more than one dimension, so there is no intrinsic way to compare two colors. You have to determine for your use case the meaning of the colors and thereby how to best compare them.

Most likely you want to compare the hue, saturation and/or lightness properties of the colors as oppposed to the red/green/blue components. If you are having trouble figuring out how you want to compare them, take some pairs of sample colors and compare them mentally, then try to justify/explain to yourself why they are similar/different.

Once you know which properties/components of the colors you want to compare, then you need to figure out how to extract that information from a color.

Most likely you will just need to convert the color from the common RedGreenBlue representation to HueSaturationLightness, and then calculate something like

avghue = (color1.hue + color2.hue)/2
distance = abs(color1.hue-avghue)

This example would give you a simple scalar value indicating how far the gradient/hue of the colors are from each other.

See HSL and HSV at Wikipedia.

CodeIgniter - how to catch DB errors?

I have created an simple library for that:

<?php
defined('BASEPATH') OR exit('No direct script access allowed');

class exceptions {

    public function checkForError() {
        get_instance()->load->database();
        $error = get_instance()->db->error();
        if ($error['code'])
            throw new MySQLException($error);
    }
}

abstract class UserException extends Exception {
    public abstract function getUserMessage();
}

class MySQLException extends UserException {
    private $errorNumber;
    private $errorMessage;

    public function __construct(array $error) {
        $this->errorNumber = "Error Code(" . $error['code'] . ")";
        $this->errorMessage = $error['message'];
    }

    public function getUserMessage() {
        return array(
            "error" => array (
                "code" => $this->errorNumber,
                "message" => $this->errorMessage
            )
        );
    }

}

The example query:

function insertId($id){
    $data = array(
        'id' => $id,
    );

    $this->db->insert('test', $data);
    $this->exceptions->checkForError();
    return $this->db->insert_id();
}

And I can catch it this way in my controller:

 try {
     $this->insertThings->insertId("1");
 } catch (UserException $error){
     //do whatever you want when there is an mysql error

 }

ALTER DATABASE failed because a lock could not be placed on database

Try this if it is "in transition" ...

http://learnmysql.blogspot.com/2012/05/database-is-in-transition-try-statement.html

USE master
GO

ALTER DATABASE <db_name>

SET OFFLINE WITH ROLLBACK IMMEDIATE
...
...
ALTER DATABASE <db_name> SET ONLINE

ERROR Source option 1.5 is no longer supported. Use 1.6 or later

You need to set JDK 1.5 to your project and also all dependent project or jar file should also compiled with JDK 1.5

How to make a input field readonly with JavaScript?

I think you just have readonly="readonly"

<html><body><form><input type="password" placeholder="password" valid="123" readonly=" readonly"></input>

Java - Find shortest path between 2 points in a distance weighted map

Estimated sanjan:

The idea behind Dijkstra's Algorithm is to explore all the nodes of the graph in an ordered way. The algorithm stores a priority queue where the nodes are ordered according to the cost from the start, and in each iteration of the algorithm the following operations are performed:

  1. Extract from the queue the node with the lowest cost from the start, N
  2. Obtain its neighbors (N') and their associated cost, which is cost(N) + cost(N, N')
  3. Insert in queue the neighbor nodes N', with the priority given by their cost

It's true that the algorithm calculates the cost of the path between the start (A in your case) and all the rest of the nodes, but you can stop the exploration of the algorithm when it reaches the goal (Z in your example). At this point you know the cost between A and Z, and the path connecting them.

I recommend you to use a library which implements this algorithm instead of coding your own. In Java, you might take a look to the Hipster library, which has a very friendly way to generate the graph and start using the search algorithms.

Here you have an example of how to define the graph and start using Dijstra with Hipster.

// Create a simple weighted directed graph with Hipster where
// vertices are Strings and edge values are just doubles
HipsterDirectedGraph<String,Double> graph = GraphBuilder.create()
  .connect("A").to("B").withEdge(4d)
  .connect("A").to("C").withEdge(2d)
  .connect("B").to("C").withEdge(5d)
  .connect("B").to("D").withEdge(10d)
  .connect("C").to("E").withEdge(3d)
  .connect("D").to("F").withEdge(11d)
  .connect("E").to("D").withEdge(4d)
  .buildDirectedGraph();

// Create the search problem. For graph problems, just use
// the GraphSearchProblem util class to generate the problem with ease.
SearchProblem p = GraphSearchProblem
  .startingFrom("A")
  .in(graph)
  .takeCostsFromEdges()
  .build();

// Search the shortest path from "A" to "F"
System.out.println(Hipster.createDijkstra(p).search("F"));

You only have to substitute the definition of the graph for your own, and then instantiate the algorithm as in the example.

I hope this helps!

Correct use of flush() in JPA/Hibernate

Can em.flush() cause any harm when using it within a transaction?

Yes, it may hold locks in the database for a longer duration than necessary.

Generally, When using JPA you delegates the transaction management to the container (a.k.a CMT - using @Transactional annotation on business methods) which means that a transaction is automatically started when entering the method and commited / rolled back at the end. If you let the EntityManager handle the database synchronization, sql statements execution will be only triggered just before the commit, leading to short lived locks in database. Otherwise your manually flushed write operations may retain locks between the manual flush and the automatic commit which can be long according to remaining method execution time.

Notes that some operation automatically triggers a flush : executing a native query against the same session (EM state must be flushed to be reachable by the SQL query), inserting entities using native generated id (generated by the database, so the insert statement must be triggered thus the EM is able to retrieve the generated id and properly manage relationships)

Java Replace Character At Specific Position Of String?

To replace a character at a specified position :

public static String replaceCharAt(String s, int pos, char c) {
   return s.substring(0,pos) + c + s.substring(pos+1);
}

How to get Android application id?

If by application id, you're referring to package name, you can use the method Context::getPackageName (http://http://developer.android.com/reference/android/content/Context.html#getPackageName%28%29).

In case you wish to communicate with other application, there are multiple ways:

  1. Start an activity of another application and send data in the "Extras" of the "Intent"
  2. Send a broadcast with specific action/category and send data in the extras
  3. If you just need to share structured data, use content provider
  4. If the other application needs to continuously run in the background, use Server and "bind" yourself to the service.

If you can elaborate your exact requirement, the community will be able to help you better.

Django Rest Framework File Upload

Use the FileUploadParser, it's all in the request. Use a put method instead, you'll find an example in the docs :)

class FileUploadView(views.APIView):
    parser_classes = (FileUploadParser,)

    def put(self, request, filename, format=None):
        file_obj = request.FILES['file']
        # do some stuff with uploaded file
        return Response(status=204)

How to remove a TFS Workspace Mapping?

If mapping is proper then you can undo/checkin your changes, if you really want to change folder name.

Alternatively if you want to remove mapping then in Visual Studio go to File-> Source Control-> Advanced-> Workspaces-> Edit

Now you can click on appropriate path and remove mapping.

Sorting an IList in C#

The accepted answer by @DavidMills is quite good, but I think it can be improved upon. For one, there is no need to define the ComparisonComparer<T> class when the framework already includes a static method Comparer<T>.Create(Comparison<T>). This method can be used to create an IComparison on the fly.

Also, it casts IList<T> to IList which has the potential to be dangerous. In most cases that I have seen, List<T> which implements IList is used behind the scenes to implement IList<T>, but this is not guaranteed and can lead to brittle code.

Lastly, the overloaded List<T>.Sort() method has 4 signatures and only 2 of them are implemented.

  1. List<T>.Sort()
  2. List<T>.Sort(Comparison<T>)
  3. List<T>.Sort(IComparer<T>)
  4. List<T>.Sort(Int32, Int32, IComparer<T>)

The below class implements all 4 List<T>.Sort() signatures for the IList<T> interface:

using System;
using System.Collections.Generic;

public static class IListExtensions
{
    public static void Sort<T>(this IList<T> list)
    {
        if (list is List<T>)
        {
            ((List<T>)list).Sort();
        }
        else
        {
            List<T> copy = new List<T>(list);
            copy.Sort();
            Copy(copy, 0, list, 0, list.Count);
        }
    }

    public static void Sort<T>(this IList<T> list, Comparison<T> comparison)
    {
        if (list is List<T>)
        {
            ((List<T>)list).Sort(comparison);
        }
        else
        {
            List<T> copy = new List<T>(list);
            copy.Sort(comparison);
            Copy(copy, 0, list, 0, list.Count);
        }
    }

    public static void Sort<T>(this IList<T> list, IComparer<T> comparer)
    {
        if (list is List<T>)
        {
            ((List<T>)list).Sort(comparer);
        }
        else
        {
            List<T> copy = new List<T>(list);
            copy.Sort(comparer);
            Copy(copy, 0, list, 0, list.Count);
        }
    }

    public static void Sort<T>(this IList<T> list, int index, int count,
        IComparer<T> comparer)
    {
        if (list is List<T>)
        {
            ((List<T>)list).Sort(index, count, comparer);
        }
        else
        {
            List<T> range = new List<T>(count);
            for (int i = 0; i < count; i++)
            {
                range.Add(list[index + i]);
            }
            range.Sort(comparer);
            Copy(range, 0, list, index, count);
        }
    }

    private static void Copy<T>(IList<T> sourceList, int sourceIndex,
        IList<T> destinationList, int destinationIndex, int count)
    {
        for (int i = 0; i < count; i++)
        {
            destinationList[destinationIndex + i] = sourceList[sourceIndex + i];
        }
    }
}

Usage:

class Foo
{
    public int Bar;

    public Foo(int bar) { this.Bar = bar; }
}

void TestSort()
{
    IList<int> ints = new List<int>() { 1, 4, 5, 3, 2 };
    IList<Foo> foos = new List<Foo>()
    {
        new Foo(1),
        new Foo(4),
        new Foo(5),
        new Foo(3),
        new Foo(2),
    };

    ints.Sort();
    foos.Sort((x, y) => Comparer<int>.Default.Compare(x.Bar, y.Bar));
}

The idea here is to leverage the functionality of the underlying List<T> to handle sorting whenever possible. Again, most IList<T> implementations that I have seen use this. In the case when the underlying collection is a different type, fallback to creating a new instance of List<T> with elements from the input list, use it to do the sorting, then copy the results back to the input list. This will work even if the input list does not implement the IList interface.

What is the naming convention in Python for variable and function names?

There is PEP 8, as other answers show, but PEP 8 is only the styleguide for the standard library, and it's only taken as gospel therein. One of the most frequent deviations of PEP 8 for other pieces of code is the variable naming, specifically for methods. There is no single predominate style, although considering the volume of code that uses mixedCase, if one were to make a strict census one would probably end up with a version of PEP 8 with mixedCase. There is little other deviation from PEP 8 that is quite as common.

Can anyone explain IEnumerable and IEnumerator to me?

The IEnumerable and IEnumerator Interfaces

To begin examining the process of implementing existing .NET interfaces, let’s first look at the role of IEnumerable and IEnumerator. Recall that C# supports a keyword named foreach that allows you to iterate over the contents of any array type:

// Iterate over an array of items.
int[] myArrayOfInts = {10, 20, 30, 40};
foreach(int i in myArrayOfInts)
{
   Console.WriteLine(i);
}

While it might seem that only array types can make use of this construct, the truth of the matter is any type supporting a method named GetEnumerator() can be evaluated by the foreach construct.To illustrate, follow me!

Suppose we have a Garage class:

// Garage contains a set of Car objects.
public class Garage
{
   private Car[] carArray = new Car[4];
   // Fill with some Car objects upon startup.
   public Garage()
   {
      carArray[0] = new Car("Rusty", 30);
      carArray[1] = new Car("Clunker", 55);
      carArray[2] = new Car("Zippy", 30);
      carArray[3] = new Car("Fred", 30);
   }
}

Ideally, it would be convenient to iterate over the Garage object’s subitems using the foreach construct, just like an array of data values:

// This seems reasonable ...
public class Program
{
   static void Main(string[] args)
   {
      Console.WriteLine("***** Fun with IEnumerable / IEnumerator *****\n");
      Garage carLot = new Garage();
      // Hand over each car in the collection?
      foreach (Car c in carLot)
      {
         Console.WriteLine("{0} is going {1} MPH",
         c.PetName, c.CurrentSpeed);
      }
      Console.ReadLine();
   }
}

Sadly, the compiler informs you that the Garage class does not implement a method named GetEnumerator(). This method is formalized by the IEnumerable interface, which is found lurking within the System.Collections namespace. Classes or structures that support this behavior advertise that they are able to expose contained subitems to the caller (in this example, the foreach keyword itself). Here is the definition of this standard .NET interface:

// This interface informs the caller
// that the object's subitems can be enumerated.
public interface IEnumerable
{
   IEnumerator GetEnumerator();
}

As you can see, the GetEnumerator() method returns a reference to yet another interface named System.Collections.IEnumerator. This interface provides the infrastructure to allow the caller to traverse the internal objects contained by the IEnumerable-compatible container:

// This interface allows the caller to
// obtain a container's subitems.
public interface IEnumerator
{
   bool MoveNext (); // Advance the internal position of the cursor.
   object Current { get;} // Get the current item (read-only property).
   void Reset (); // Reset the cursor before the first member.
}

If you want to update the Garage type to support these interfaces, you could take the long road and implement each method manually. While you are certainly free to provide customized versions of GetEnumerator(), MoveNext(), Current, and Reset(), there is a simpler way. As the System.Array type (as well as many other collection classes) already implements IEnumerable and IEnumerator, you can simply delegate the request to the System.Array as follows:

using System.Collections;
...
public class Garage : IEnumerable
{
   // System.Array already implements IEnumerator!
   private Car[] carArray = new Car[4];
   public Garage()
   {
      carArray[0] = new Car("FeeFee", 200);
      carArray[1] = new Car("Clunker", 90);
      carArray[2] = new Car("Zippy", 30);
      carArray[3] = new Car("Fred", 30);
   }
   public IEnumerator GetEnumerator()
   {
      // Return the array object's IEnumerator.
      return carArray.GetEnumerator();
   }
}

After you have updated your Garage type, you can safely use the type within the C# foreach construct. Furthermore, given that the GetEnumerator() method has been defined publicly, the object user could also interact with the IEnumerator type:

// Manually work with IEnumerator.
IEnumerator i = carLot.GetEnumerator();
i.MoveNext();
Car myCar = (Car)i.Current;
Console.WriteLine("{0} is going {1} MPH", myCar.PetName, myCar.CurrentSpeed);

However, if you prefer to hide the functionality of IEnumerable from the object level, simply make use of explicit interface implementation:

IEnumerator IEnumerable.GetEnumerator()
{
  // Return the array object's IEnumerator.
  return carArray.GetEnumerator();
}

By doing so, the casual object user will not find the Garage’s GetEnumerator() method, while the foreach construct will obtain the interface in the background when necessary.

Adapted from the Pro C# 5.0 and the .NET 4.5 Framework

Substitute a comma with a line break in a cell

Windows (unlike some other OS's, like Linux), uses CR+LF for line breaks:

  • CR = 13 = 0x0D = ^M = \r = carriage return

  • LF = 10 = 0x0A = ^J = \n = new line

The characters need to be in that order, if you want the line breaks to be consistently visible when copied to other Windows programs. So the Excel function would be:

=SUBSTITUTE(A1,",",CHAR(13) & CHAR(10))

Can two or more people edit an Excel document at the same time?

yes if it is SharePoint 2010 and above by using the Office feature co-authoring

Create a map with clickable provinces/states using SVG, HTML/CSS, ImageMap

The following code may help you:

$("#svgEuropa [id='stallwanger.it.dev_shape_DEU']").on("click",function(){
    alert($(this).attr("id"));
});

Source

How to disable compiler optimizations in gcc?

For gcc you want to omit any -O1 -O2 or -O3 options passed to the compiler or if you already have them you can append the -O0 option to turn it off again. It might also help you to add -g for debug so that you can see the c source and disassembled machine code in your debugger.

See also: http://sourceware.org/gdb/onlinedocs/gdb/Optimized-Code.html

In C#, why is String a reference type that behaves like a value type?

The distinction between reference types and value types are basically a performance tradeoff in the design of the language. Reference types have some overhead on construction and destruction and garbage collection, because they are created on the heap. Value types on the other hand have overhead on method calls (if the data size is larger than a pointer), because the whole object is copied rather than just a pointer. Because strings can be (and typically are) much larger than the size of a pointer, they are designed as reference types. Also, as Servy pointed out, the size of a value type must be known at compile time, which is not always the case for strings.

The question of mutability is a separate issue. Both reference types and value types can be either mutable or immutable. Value types are typically immutable though, since the semantics for mutable value types can be confusing.

Reference types are generally mutable, but can be designed as immutable if it makes sense. Strings are defined as immutable because it makes certain optimizations possible. For example, if the same string literal occurs multiple times in the same program (which is quite common), the compiler can reuse the same object.

So why is "==" overloaded to compare strings by text? Because it is the most useful semantics. If two strings are equal by text, they may or may not be the same object reference due to the optimizations. So comparing references are pretty useless, while comparing text are almost always what you want.

Speaking more generally, Strings has what is termed value semantics. This is a more general concept than value types, which is a C# specific implementation detail. Value types have value semantics, but reference types may also have value semantics. When a type have value semantics, you can't really tell if the underlying implementation is a reference type or value type, so you can consider that an implementation detail.

is vs typeof

They don't do the same thing. The first one works if obj is of type ClassA or of some subclass of ClassA. The second one will only match objects of type ClassA. The second one will be faster since it doesn't have to check the class hierarchy.

For those who want to know the reason, but don't want to read the article referenced in is vs typeof.

Python PIP Install throws TypeError: unsupported operand type(s) for -=: 'Retry' and 'int'

If you are behind a proxy, you must do some extra configuration steps before starting the installation. You must set the environment variable http_proxy to the proxy address. Using bash this is accomplished with the command

export http_proxy="http://user:[email protected]:port/" 

You can also provide the

--proxy=[user:pass@]url:port 

parameter to pip. The [user:pass@] portion is optional.

Android: how to make an activity return results to the activity which calls it?

In order to start an activity which should return result to the calling activity, you should do something like below. You should pass the requestcode as shown below in order to identify that you got the result from the activity you started.

startActivityForResult(new Intent(“YourFullyQualifiedClassName”),requestCode);

In the activity you can make use of setData() to return result.

Intent data = new Intent();
String text = "Result to be returned...."
//---set the data to pass back---
data.setData(Uri.parse(text));
setResult(RESULT_OK, data);
//---close the activity---
finish();

So then again in the first activity you write the below code in onActivityResult()

public void onActivityResult(int requestCode, int resultCode, Intent data) {
    if (requestCode == request_Code) {
        if (resultCode == RESULT_OK) {
            String returnedResult = data.getData().toString();
            // OR
            // String returnedResult = data.getDataString();
        }
    }
}

EDIT based on your comment: If you want to return three strings, then follow this by making use of key/value pairs with intent instead of using Uri.

Intent data = new Intent();
data.putExtra("streetkey","streetname");
data.putExtra("citykey","cityname");
data.putExtra("homekey","homename");
setResult(RESULT_OK,data);
finish();

Get them in onActivityResult like below:

public void onActivityResult(int requestCode, int resultCode, Intent data) {
    if (requestCode == request_Code) {
        if (resultCode == RESULT_OK) {
            String street = data.getStringExtra("streetkey");
            String city = data.getStringExtra("citykey");
            String home = data.getStringExtra("homekey");
        }
    }
}

jquery change div text

Put the title in its own span.

<span id="dialog_title_span">'+dialog_title+'</span>
$('#dialog_title_span').text("new dialog title");

Iterating through a List Object in JSP

 <c:forEach items="${sessionScope.empL}" var="emp">
            <tr>
                <td>Employee ID: <c:out value="${emp.eid}"/></td>
                <td>Employee Pass: <c:out value="${emp.ename}"/></td>  
            </tr>
        </c:forEach>

sort files by date in PHP

An example that uses RecursiveDirectoryIterator class, it's a convenient way to iterate recursively over filesystem.

$output = array();
foreach( new RecursiveIteratorIterator( 
    new RecursiveDirectoryIterator( 'path', FilesystemIterator::SKIP_DOTS | FilesystemIterator::UNIX_PATHS ) ) as $value ) {      
        if ( $value->isFile() ) {
            $output[] = array( $value->getMTime(), $value->getRealPath() );
        }
}

usort ( $output, function( $a, $b ) {
    return $a[0] > $b[0];
});

Spring MVC @PathVariable with dot (.) is getting truncated

adding the ":.+" worked for me, but not until I removed outer curly brackets.

value = {"/username/{id:.+}"} didn't work

value = "/username/{id:.+}" works

Hope I helped someone :)

How do I count a JavaScript object's attributes?

You can do that by using this simple code:

Object.keys(myObject).length

Is there a good JSP editor for Eclipse?

I followed the advice of Simon Gibbs in this answer and found it worked out fine - if you're in a hurry, the "Web Page Editor (optional)" package from the Eclipse update site does the trick.

For the Eclipse-challenged (me) Help > Install New Software > Work with > Expand Web, XML, and Java EE Development > Select "Web Page Editor (optional)" and "next-through" to completion.

javascript check for not null

It's because val is not null, but contains 'null' as a string.

Try to check with 'null'

if ('null' != val)

For an explanation of when and why this works, see the details below.

Using the GET parameter of a URL in JavaScript

This looked ok:

function gup( name ){
   name = name.replace(/[\[]/,"\\\[").replace(/[\]]/,"\\\]");
   var regexS = "[\\?&]"+name+"=([^&#]*)";
   var regex = new RegExp( regexS );
   var results = regex.exec( window.location.href );
   if( results == null )
      return "";
   else
      return results[1];
}

From http://www.netlobo.com/url_query_string_javascript.html

Target WSGI script cannot be loaded as Python module

I know this question is pretty old, but I wrestled with this for about eight hours just now. If you have a system with SELinux enabled and you've put your virtualenv in particular places, mod_wsgi won't be able to add your specified python-path to the site-packages. It also won't raise any errors; as it turns out the mechanism it uses to add the specified python-path to the site packages is with the Python site module, specifically site.adduserdir(). This method doesn't raise any errors if the directory is missing or can't be accessed, so mod_wsgi also doesn't raise any errors.

Anyway, try turning off SELinux with

sudo setenforce 0

or by making sure that the process you're running Apache as has the appropriate ACLs with SELinux to access the directory the virtualenv is in.

Angular 5, HTML, boolean on checkbox is checked

Work with checkboxes using observables

You could even choose to use a behaviourSubject to utilize the power of observables so you can start a certain chain of reaction starting at the isChecked$ observable.

In your component.ts:

public isChecked$ = new BehaviorSubject(false);
toggleChecked() {
  this.isChecked$.next(!this.isChecked$.value)
}

In your template

<input type="checkbox" [checked]="isChecked$ | async" (change)="toggleChecked()">

What exactly is Python's file.flush() doing?

Basically, flush() cleans out your RAM buffer, its real power is that it lets you continue to write to it afterwards - but it shouldn't be thought of as the best/safest write to file feature. It's flushing your RAM for more data to come, that is all. If you want to ensure data gets written to file safely then use close() instead.

Use cell's color as condition in if statement (function)

You cannot use VBA (Interior.ColorIndex) in a formula which is why you receive the error.

It is not possible to do this without VBA.

Function YellowIt(rng As Range) As Boolean
    If rng.Interior.ColorIndex = 6 Then
        YellowIt = True
    Else
        YellowIt = False
    End If
End Function

However, I do not recommend this: it is not how user-defined VBA functions (UDFs) are intended to be used. They should reflect the behaviour of Excel functions, which cannot read the colour-formatting of a cell. (This function may not work in a future version of Excel.)

It is far better that you base a formula on the original condition (decision) that makes the cell yellow in the first place. Or, alternatively, run a Sub procedure to fill in the True or False values (although, of course, these values will no longer be linked to the original cell's formatting).

How to sort by column in descending order in Spark SQL?

import org.apache.spark.sql.functions.desc

df.orderBy(desc("columnname1"),desc("columnname2"),asc("columnname3"))

How do you change the server header returned by nginx?

I know the post is kinda old, but I have found a solution easy that works on Debian based distribution without compiling nginx from source.

First install nginx-extras package

sudo apt install nginx-extras

Then load the nginx http headers more module by editing nginx.conf and adding the following line inside the server block

load_module modules/ngx_http_headers_more_filter_module.so;

Once it's done you'll have access to both more_set_headers and more_clear_headers directives.

Best way to get hostname with php

I am running PHP version 5.4 on shared hosting and both of these both successfully return the same results:

php_uname('n');

gethostname();

How do I use arrays in C++?

Array creation and initialization

As with any other kind of C++ object, arrays can be stored either directly in named variables (then the size must be a compile-time constant; C++ does not support VLAs), or they can be stored anonymously on the heap and accessed indirectly via pointers (only then can the size be computed at runtime).

Automatic arrays

Automatic arrays (arrays living "on the stack") are created each time the flow of control passes through the definition of a non-static local array variable:

void foo()
{
    int automatic_array[8];
}

Initialization is performed in ascending order. Note that the initial values depend on the element type T:

  • If T is a POD (like int in the above example), no initialization takes place.
  • Otherwise, the default-constructor of T initializes all the elements.
  • If T provides no accessible default-constructor, the program does not compile.

Alternatively, the initial values can be explicitly specified in the array initializer, a comma-separated list surrounded by curly brackets:

    int primes[8] = {2, 3, 5, 7, 11, 13, 17, 19};

Since in this case the number of elements in the array initializer is equal to the size of the array, specifying the size manually is redundant. It can automatically be deduced by the compiler:

    int primes[] = {2, 3, 5, 7, 11, 13, 17, 19};   // size 8 is deduced

It is also possible to specify the size and provide a shorter array initializer:

    int fibonacci[50] = {0, 1, 1};   // 47 trailing zeros are deduced

In that case, the remaining elements are zero-initialized. Note that C++ allows an empty array initializer (all elements are zero-initialized), whereas C89 does not (at least one value is required). Also note that array initializers can only be used to initialize arrays; they cannot later be used in assignments.

Static arrays

Static arrays (arrays living "in the data segment") are local array variables defined with the static keyword and array variables at namespace scope ("global variables"):

int global_static_array[8];

void foo()
{
    static int local_static_array[8];
}

(Note that variables at namespace scope are implicitly static. Adding the static keyword to their definition has a completely different, deprecated meaning.)

Here is how static arrays behave differently from automatic arrays:

  • Static arrays without an array initializer are zero-initialized prior to any further potential initialization.
  • Static POD arrays are initialized exactly once, and the initial values are typically baked into the executable, in which case there is no initialization cost at runtime. This is not always the most space-efficient solution, however, and it is not required by the standard.
  • Static non-POD arrays are initialized the first time the flow of control passes through their definition. In the case of local static arrays, that may never happen if the function is never called.

(None of the above is specific to arrays. These rules apply equally well to other kinds of static objects.)

Array data members

Array data members are created when their owning object is created. Unfortunately, C++03 provides no means to initialize arrays in the member initializer list, so initialization must be faked with assignments:

class Foo
{
    int primes[8];

public:

    Foo()
    {
        primes[0] = 2;
        primes[1] = 3;
        primes[2] = 5;
        // ...
    }
};

Alternatively, you can define an automatic array in the constructor body and copy the elements over:

class Foo
{
    int primes[8];

public:

    Foo()
    {
        int local_array[] = {2, 3, 5, 7, 11, 13, 17, 19};
        std::copy(local_array + 0, local_array + 8, primes + 0);
    }
};

In C++0x, arrays can be initialized in the member initializer list thanks to uniform initialization:

class Foo
{
    int primes[8];

public:

    Foo() : primes { 2, 3, 5, 7, 11, 13, 17, 19 }
    {
    }
};

This is the only solution that works with element types that have no default constructor.

Dynamic arrays

Dynamic arrays have no names, hence the only means of accessing them is via pointers. Because they have no names, I will refer to them as "anonymous arrays" from now on.

In C, anonymous arrays are created via malloc and friends. In C++, anonymous arrays are created using the new T[size] syntax which returns a pointer to the first element of an anonymous array:

std::size_t size = compute_size_at_runtime();
int* p = new int[size];

The following ASCII art depicts the memory layout if the size is computed as 8 at runtime:

             +---+---+---+---+---+---+---+---+
(anonymous)  |   |   |   |   |   |   |   |   |
             +---+---+---+---+---+---+---+---+
               ^
               |
               |
             +-|-+
          p: | | |                               int*
             +---+

Obviously, anonymous arrays require more memory than named arrays due to the extra pointer that must be stored separately. (There is also some additional overhead on the free store.)

Note that there is no array-to-pointer decay going on here. Although evaluating new int[size] does in fact create an array of integers, the result of the expression new int[size] is already a pointer to a single integer (the first element), not an array of integers or a pointer to an array of integers of unknown size. That would be impossible, because the static type system requires array sizes to be compile-time constants. (Hence, I did not annotate the anonymous array with static type information in the picture.)

Concerning default values for elements, anonymous arrays behave similar to automatic arrays. Normally, anonymous POD arrays are not initialized, but there is a special syntax that triggers value-initialization:

int* p = new int[some_computed_size]();

(Note the trailing pair of parenthesis right before the semicolon.) Again, C++0x simplifies the rules and allows specifying initial values for anonymous arrays thanks to uniform initialization:

int* p = new int[8] { 2, 3, 5, 7, 11, 13, 17, 19 };

If you are done using an anonymous array, you have to release it back to the system:

delete[] p;

You must release each anonymous array exactly once and then never touch it again afterwards. Not releasing it at all results in a memory leak (or more generally, depending on the element type, a resource leak), and trying to release it multiple times results in undefined behavior. Using the non-array form delete (or free) instead of delete[] to release the array is also undefined behavior.

How to declare an array of strings in C++?

Instead of that macro, might I suggest this one:

template<typename T, int N>
inline size_t array_size(T(&)[N])
{
    return N;
}

#define ARRAY_SIZE(X)   (sizeof(array_size(X)) ? (sizeof(X) / sizeof((X)[0])) : -1)

1) We want to use a macro to make it a compile-time constant; the function call's result is not a compile-time constant.

2) However, we don't want to use a macro because the macro could be accidentally used on a pointer. The function can only be used on compile-time arrays.

So, we use the defined-ness of the function to make the macro "safe"; if the function exists (i.e. it has non-zero size) then we use the macro as above. If the function does not exist we return a bad value.

Git cli: get user info from username

git config --list

git config -l

will display your username and email together, along with other info

How to sort Counter by value? - python

Use the Counter.most_common() method, it'll sort the items for you:

>>> from collections import Counter
>>> x = Counter({'a':5, 'b':3, 'c':7})
>>> x.most_common()
[('c', 7), ('a', 5), ('b', 3)]

It'll do so in the most efficient manner possible; if you ask for a Top N instead of all values, a heapq is used instead of a straight sort:

>>> x.most_common(1)
[('c', 7)]

Outside of counters, sorting can always be adjusted based on a key function; .sort() and sorted() both take callable that lets you specify a value on which to sort the input sequence; sorted(x, key=x.get, reverse=True) would give you the same sorting as x.most_common(), but only return the keys, for example:

>>> sorted(x, key=x.get, reverse=True)
['c', 'a', 'b']

or you can sort on only the value given (key, value) pairs:

>>> sorted(x.items(), key=lambda pair: pair[1], reverse=True)
[('c', 7), ('a', 5), ('b', 3)]

See the Python sorting howto for more information.

Sending images using Http Post

Version 4.3.5 Updated Code

  • httpclient-4.3.5.jar
  • httpcore-4.3.2.jar
  • httpmime-4.3.5.jar

Since MultipartEntity has been deprecated. Please see the code below.

String responseBody = "failure";
HttpClient client = new DefaultHttpClient();
client.getParams().setParameter(CoreProtocolPNames.PROTOCOL_VERSION, HttpVersion.HTTP_1_1);

String url = WWPApi.URL_USERS;
Map<String, String> map = new HashMap<String, String>();
map.put("user_id", String.valueOf(userId));
map.put("action", "update");
url = addQueryParams(map, url);

HttpPost post = new HttpPost(url);
post.addHeader("Accept", "application/json");

MultipartEntityBuilder builder = MultipartEntityBuilder.create();
builder.setCharset(MIME.UTF8_CHARSET);

if (career != null)
    builder.addTextBody("career", career, ContentType.create("text/plain", MIME.UTF8_CHARSET));
if (gender != null)
    builder.addTextBody("gender", gender, ContentType.create("text/plain", MIME.UTF8_CHARSET));
if (username != null)
    builder.addTextBody("username", username, ContentType.create("text/plain", MIME.UTF8_CHARSET));
if (email != null)
    builder.addTextBody("email", email, ContentType.create("text/plain", MIME.UTF8_CHARSET));
if (password != null)
    builder.addTextBody("password", password, ContentType.create("text/plain", MIME.UTF8_CHARSET));
if (country != null)
    builder.addTextBody("country", country, ContentType.create("text/plain", MIME.UTF8_CHARSET));
if (file != null)
    builder.addBinaryBody("Filedata", file, ContentType.MULTIPART_FORM_DATA, file.getName());

post.setEntity(builder.build());

try {
    responseBody = EntityUtils.toString(client.execute(post).getEntity(), "UTF-8");
//  System.out.println("Response from Server ==> " + responseBody);

    JSONObject object = new JSONObject(responseBody);
    Boolean success = object.optBoolean("success");
    String message = object.optString("error");

    if (!success) {
        responseBody = message;
    } else {
        responseBody = "success";
    }

} catch (Exception e) {
    e.printStackTrace();
} finally {
    client.getConnectionManager().shutdown();
}

Naming returned columns in Pandas aggregate function?

With the inspiration of @Joel Ostblom

For those who already have a workable dictionary for merely aggregation, you can use/modify the following code for the newer version aggregation, separating aggregation and renaming part. Please be aware of the nested dictionary if there are more than 1 item.

def agg_translate_agg_rename(input_agg_dict):
    agg_dict = {}
    rename_dict = {}
    for k, v in input_agg_dict.items():
        if len(v) == 1:
            agg_dict[k] = list(v.values())[0]
            rename_dict[k] = list(v.keys())[0]
        else:
            updated_index = 1
            for nested_dict_k, nested_dict_v in v.items():
                modified_key = k + "_" + str(updated_index)
                agg_dict[modified_key] = nested_dict_v
                rename_dict[modified_key] = nested_dict_k
                updated_index += 1
    return agg_dict, rename_dict

one_dict = {"column1": {"foo": 'sum'}, "column2": {"mean": 'mean', "std": 'std'}}
agg, rename = agg_translator_aa(one_dict)

We get

agg = {'column1': 'sum', 'column2_1': 'mean', 'column2_2': 'std'}
rename = {'column1': 'foo', 'column2_1': 'mean', 'column2_2': 'std'}

Please let me know if there is a smarter way to do it. Thanks.

Compress images on client side before uploading

I read about an experiment here: http://webreflection.blogspot.com/2010/12/100-client-side-image-resizing.html

The theory is that you can use canvas to resize the images on the client before uploading. The prototype example seems to work only in recent browsers, interesting idea though...

However, I’m not sure about using canvas to compress images, but you can certainly resize them.

Using "If cell contains #N/A" as a formula condition.

You can also use IFNA(expression, value)

Why are exclamation marks used in Ruby methods?

From themomorohoax.com:

A bang can used in the below ways, in order of my personal preference.

1) An active record method raises an error if the method does not do what it says it will.

2) An active record method saves the record or a method saves an object (e.g. strip!)

3) A method does something “extra”, like posts to someplace, or does some action.

The point is: only use a bang when you’ve really thought about whether it’s necessary, to save other developers the annoyance of having to check why you are using a bang.

The bang provides two cues to other developers.

1) that it’s not necessary to save the object after calling the method.

2) when you call the method, the db is going to be changed.

http://www.themomorohoax.com/2009/02/11/when-to-use-a-bang-exclamation-point-after-rails-methods

How do I toggle an ng-show in AngularJS based on a boolean?

If you have multiple Menus with Submenus, then you can go with the below solution.

HTML

          <ul class="sidebar-menu" id="nav-accordion">
             <li class="sub-menu">
                  <a href="" ng-click="hasSubMenu('dashboard')">
                      <i class="fa fa-book"></i>
                      <span>Dashboard</span>
                      <i class="fa fa-angle-right pull-right"></i>
                  </a>
                  <ul class="sub" ng-show="showDash">
                      <li><a ng-class="{ active: isActive('/dashboard/loan')}" href="#/dashboard/loan">Loan</a></li>
                      <li><a ng-class="{ active: isActive('/dashboard/recovery')}" href="#/dashboard/recovery">Recovery</a></li>
                  </ul>
              </li>
              <li class="sub-menu">
                  <a href="" ng-click="hasSubMenu('customerCare')">
                      <i class="fa fa-book"></i>
                      <span>Customer Care</span>
                      <i class="fa fa-angle-right pull-right"></i>
                  </a>
                  <ul class="sub" ng-show="showCC">
                      <li><a ng-class="{ active: isActive('/customerCare/eligibility')}" href="#/CC/eligibility">Eligibility</a></li>
                      <li><a ng-class="{ active: isActive('/customerCare/transaction')}" href="#/CC/transaction">Transaction</a></li>
                  </ul>
              </li>
          </ul>

There are two functions i have called first is ng-click = hasSubMenu('dashboard'). This function will be used to toggle the menu and it is explained in the code below. The ng-class="{ active: isActive('/customerCare/transaction')} it will add a class active to the current menu item.

Now i have defined some functions in my app:

First, add a dependency $rootScope which is used to declare variables and functions. To learn more about $roootScope refer to the link : https://docs.angularjs.org/api/ng/service/$rootScope

Here is my app file:

 $rootScope.isActive = function (viewLocation) { 
                return viewLocation === $location.path();
        };

The above function is used to add active class to the current menu item.

        $rootScope.showDash = false;
        $rootScope.showCC = false;

        var location = $location.url().split('/');

        if(location[1] == 'customerCare'){
            $rootScope.showCC = true;
        }
        else if(location[1]=='dashboard'){
            $rootScope.showDash = true;
        }

        $rootScope.hasSubMenu = function(menuType){
            if(menuType=='dashboard'){
                $rootScope.showCC = false;
                $rootScope.showDash = $rootScope.showDash === false ? true: false;
            }
            else if(menuType=='customerCare'){
                $rootScope.showDash = false;
                $rootScope.showCC = $rootScope.showCC === false ? true: false;
            }
        }

By default $rootScope.showDash and $rootScope.showCC are set to false. It will set the menus to closed when page is initially loaded. If you have more than two submenus add accordingly.

hasSubMenu() function will work for toggling between the menus. I have added a small condition

if(location[1] == 'customerCare'){
                $rootScope.showCC = true;
            }
            else if(location[1]=='dashboard'){
                $rootScope.showDash = true;
            }

it will remain the submenu open after reloading the page according to selected menu item.

I have defined my pages like:

$routeProvider
        .when('/dasboard/loan', {
            controller: 'LoanController',
            templateUrl: './views/loan/view.html',
            controllerAs: 'vm'
        })

You can use isActive() function only if you have a single menu without submenu. You can modify the code according to your requirement. Hope this will help. Have a great day :)

Change the image source on rollover using jQuery

<img src="img1.jpg" data-swap="img2.jpg"/>



img = {

 init: function() {
  $('img').on('mouseover', img.swap);
  $('img').on('mouseover', img.swap);
 }, 

 swap: function() {
  var tmp = $(this).data('swap');
  $(this).attr('data-swap', $(this).attr('src'));
  $(this).attr('str', tmp);
 }
}

img.init();

How do I replace a character at a particular index in JavaScript?

Lets say you want to replace Kth index (0-based index) with 'Z'. You could use Regex to do this.

var re = var re = new RegExp("((.){" + K + "})((.){1})")
str.replace(re, "$1A$`");

How to get a list of all valid IP addresses in a local network?

If you want to see which IP addresses are in use on a specific subnet then there are several different IP Address managers.

Try Angry IP Scanner or Solarwinds or Advanced IP Scanner

Get query from java.sql.PreparedStatement

A bit of a hack, but it works fine for me:

Integer id = 2;
String query = "SELECT * FROM table WHERE id = ?";
PreparedStatement statement = m_connection.prepareStatement( query );
statement.setObject( 1, value );
String statementText = statement.toString();
query = statementText.substring( statementText.indexOf( ": " ) + 2 );

YouTube Autoplay not working

This code allows you to autoplay iframe video

<iframe src="https://www.youtube.com/embed/2MpUj-Aua48?rel=0&modestbranding=1&autohide=1&mute=1&showinfo=0&controls=0&autoplay=1"  width="560" height="315"  frameborder="0" allowfullscreen></iframe>

Here Is working fiddle

What to do on TransactionTooLargeException

I found the root cause of this (we got both "adding window failed" and file descriptor leak as mvds says).

There is a bug in BitmapFactory.decodeFileDescriptor() of Android 4.4. It only occurs when inPurgeable and inInputShareable of BitmapOptions are set to true. This causes many problem in many places interact with files.

Note that the method is also called from MediaStore.Images.Thumbnails.getThumbnail().

Universal Image Loader is affected by this issue. Picasso and Glide seems to be not affected. https://github.com/nostra13/Android-Universal-Image-Loader/issues/1020

How do I find Waldo with Mathematica?

I have a quick solution for finding Waldo using OpenCV.

I used the template matching function available in OpenCV to find Waldo.

To do this a template is needed. So I cropped Waldo from the original image and used it as a template.

enter image description here

Next I called the cv2.matchTemplate() function along with the normalized correlation coefficient as the method used. It returned a high probability at a single region as shown in white below (somewhere in the top left region):

enter image description here

The position of the highest probable region was found using cv2.minMaxLoc() function, which I then used to draw the rectangle to highlight Waldo:

enter image description here

ng-mouseover and leave to toggle item using mouse in angularjs

Angular solution

You can fix it like this:

$scope.hoverIn = function(){
    this.hoverEdit = true;
};

$scope.hoverOut = function(){
    this.hoverEdit = false;
};

Inside of ngMouseover (and similar) functions context is a current item scope, so this refers to the current child scope.

Also you need to put ngRepeat on li:

<ul>
    <li ng-repeat="task in tasks" ng-mouseover="hoverIn()" ng-mouseleave="hoverOut()">
        {{task.name}}
        <span ng-show="hoverEdit">
            <a>Edit</a>
        </span>
    </li>
</ul>

Demo

CSS solution

However, when possible try to do such things with CSS only, this would be the optimal solution and no JS required:

ul li span {display: none;}
ul li:hover span {display: inline;}

Lua String replace

Try:

name = "^aH^ai"
name = name:gsub("%^a", "")

See also: http://lua-users.org/wiki/StringLibraryTutorial

Find in Files: Search all code in Team Foundation Server

There is another alternative solution, that seems to be more attractive.

  1. Setup a search server - could be any windows machine/server
  2. Setup a TFS notification service* (Bissubscribe) to get, delete, update files everytime a checkin happens. So this is a web service that acts like a listener on the TFS server, and updates/syncs the files and folders on the Search server. - this will dramatically improve the accuracy (live search), and avoid the one-time load of making periodic gets
  3. Setup an indexing service/windows indexed search on the Search server for the root folder
  4. Expose a web service to return search results

Now with all the above setup, you have a few options for the client:

  1. Setup a web page to call the search service and format the results to show on the webpage - you can also integrate this webpage inside visual studio (through a macro or a add-in)
  2. Create a windows client interface(winforms/wpf) to call the search service and format the results and show them on the UI - you can also integrate this client tool inside visual studio via VSPackages or add-in

Update: I did go this route, and it has been working nicely. Just wanted to add to this.

Reference links:

  1. Use this tool instead of bissubscribe.exe
  2. Handling TFS events
  3. Team System Notifications

Android Fragments and animation

I solve this the way Below

Animation anim = AnimationUtils.loadAnimation(this, R.anim.slide);
fg.startAnimation(anim);
this.fg.setVisibility(View.VISIBLE); //fg is a View object indicate fragment

How to list physical disks?

If you only need to look at the existing disks, this one will suffice:

powershell "get-physicaldisk"

Add item to Listview control

The ListView control uses the Items collection to add items to listview in the control and is able to customize items.

Get the height and width of the browser viewport without scrollbars using jquery?

$(window).height();
$(window).width();

More info

Using jQuery is not essential for getting those values, however. Use

document.documentElement.clientHeight;
document.documentElement.clientWidth;

to get sizes excluding scrollbars, or

window.innerHeight;
window.innerWidth;

to get the whole viewport, including scrollbars.

document.documentElement.clientHeight <= window.innerHeight;  // is always true

Exception 'open failed: EACCES (Permission denied)' on Android

It turned out, it was a stupid mistake since I had my phone still connected to the desktop PC and didn't realize this.

So I had to turn off the USB connection and everything worked fine.

Getting the last element of a split string array

There's a one-liner for everything. :)

var output = input.split(/[, ]+/).pop();

Split files using tar, gz, zip, or bzip2

You can use the split command with the -b option:

split -b 1024m file.tar.gz

It can be reassembled on a Windows machine using @Joshua's answer.

copy /b file1 + file2 + file3 + file4 filetogether

Edit: As @Charlie stated in the comment below, you might want to set a prefix explicitly because it will use x otherwise, which can be confusing.

split -b 1024m "file.tar.gz" "file.tar.gz.part-"

// Creates files: file.tar.gz.part-aa, file.tar.gz.part-ab, file.tar.gz.part-ac, ...

Edit: Editing the post because question is closed and the most effective solution is very close to the content of this answer:

# create archives
$ tar cz my_large_file_1 my_large_file_2 | split -b 1024MiB - myfiles_split.tgz_
# uncompress
$ cat myfiles_split.tgz_* | tar xz

This solution avoids the need to use an intermediate large file when (de)compressing. Use the tar -C option to use a different directory for the resulting files. btw if the archive consists from only a single file, tar could be avoided and only gzip used:

# create archives
$ gzip -c my_large_file | split -b 1024MiB - myfile_split.gz_
# uncompress
$ cat myfile_split.gz_* | gunzip -c > my_large_file

For windows you can download ported versions of the same commands or use cygwin.

How to use GROUP BY to concatenate strings in SQL Server?

If you have clr enabled you could use the Group_Concat library from GitHub

Regular expression to match characters at beginning of line only

^CTR

or

^CTR.*

edit:

To be more clear: ^CTR will match start of line and those chars. If all you want to do is match for a line itself (and already have the line to use), then that is all you really need. But if this is the case, you may be better off using a prefab substr() type function. I don't know, what language are you are using. But if you are trying to match and grab the line, you will need something like .* or .*$ or whatever, depending on what language/regex function you are using.

Recommended way to get hostname in Java

Environment variables may also provide a useful means -- COMPUTERNAME on Windows, HOSTNAME on most modern Unix/Linux shells.

See: https://stackoverflow.com/a/17956000/768795

I'm using these as "supplementary" methods to InetAddress.getLocalHost().getHostName(), since as several people point out, that function doesn't work in all environments.

Runtime.getRuntime().exec("hostname") is another possible supplement. At this stage, I haven't used it.

import java.net.InetAddress;
import java.net.UnknownHostException;

// try InetAddress.LocalHost first;
//      NOTE -- InetAddress.getLocalHost().getHostName() will not work in certain environments.
try {
    String result = InetAddress.getLocalHost().getHostName();
    if (StringUtils.isNotEmpty( result))
        return result;
} catch (UnknownHostException e) {
    // failed;  try alternate means.
}

// try environment properties.
//      
String host = System.getenv("COMPUTERNAME");
if (host != null)
    return host;
host = System.getenv("HOSTNAME");
if (host != null)
    return host;

// undetermined.
return null;

How can I do time/hours arithmetic in Google Spreadsheet?

So much simpler: look at this

B2: 23:00
C2:  1:37
D2: = C2 - B2 + ( B2 > C2 )

Why it works, time is a fraction of a day, the comparison B2>C2 returns True (1) or False (0), if true 1 day (24 hours) is added. http://www.excelforum.com/excel-general/471757-calculating-time-difference-over-midnight.html

Cannot attach the file *.mdf as database

Remove this line from the connection string that should do it ;) "AttachDbFilename=|DataDirectory|whateverurdatabasenameis-xxxxxxxxxx.mdf"

How do you count the lines of code in a Visual Studio solution?

I used Ctrl+Shift+F. Next, put a \n in the search box and enable regular expressions box. Then in the find results, in the end of the screen are the number of files searched and lines of code found.

You can use [^\n\s]\r\n to skip blank and space-only lines (credits to Zach in the comments).

Is there a Visual Basic 6 decompiler?

Yes I think You can get it download and separately its Help files from: vbdecompiler.org Site. and there is a Video on YouTube which explains how to Use it to Get the Code from an exe file and Save it. I hope that I helped.

Pure JavaScript equivalent of jQuery's $.ready() - how to call a function when the page/DOM is ready for it

The good folks at HubSpot have a resource where you can find pure Javascript methodologies for achieving a lot of jQuery goodness - including ready

http://youmightnotneedjquery.com/#ready

function ready(fn) {
  if (document.readyState != 'loading'){
    fn();
  } else if (document.addEventListener) {
    document.addEventListener('DOMContentLoaded', fn);
  } else {
    document.attachEvent('onreadystatechange', function() {
      if (document.readyState != 'loading')
        fn();
    });
  }
}

example inline usage:

ready(function() { alert('hello'); });

How to send an HTTP request using Telnet

For posterity, your question was how to send an http request to https://stackoverflow.com/questions. The real answer is: you cannot with telnet, cause this is an https-only reachable url.

So, you might want to use openssl instead of telnet, like this for instance

$ openssl s_client -connect stackoverflow.com:443
...
---
GET /questions HTTP/1.1
Host: stackoverflow.com

This will give you the https response.

Batch file to map a drive when the folder name contains spaces

I just created some directories, shared them and mapped using:

net use y: "\\mycomputername\folder with spaces"

So this solution gets "works on my machine" certificate. What error code do you get?

How to add users to Docker container?

The trick is to use useradd instead of its interactive wrapper adduser. I usually create users with:

RUN useradd -ms /bin/bash newuser

which creates a home directory for the user and ensures that bash is the default shell.

You can then add:

USER newuser
WORKDIR /home/newuser

to your dockerfile. Every command afterwards as well as interactive sessions will be executed as user newuser:

docker run -t -i image
newuser@131b7ad86360:~$

You might have to give newuser the permissions to execute the programs you intend to run before invoking the user command.

Using non-privileged users inside containers is a good idea for security reasons. It also has a few drawbacks. Most importantly, people deriving images from your image will have to switch back to root before they can execute commands with superuser privileges.

How to get Last record from Sqlite?

I wanted to maintain my table while pulling in one row that gives me the last value in a particular column in the table. I essentially was looking to replace the LAST() function in excel and this worked.

, (Select column_name FROM report WHERE rowid = (select last_insert_rowid() from report))

How to run cron job every 2 hours

0 */2 * * *

The answer is from https://crontab.guru/every-2-hours. It is interesting.

Disabling the button after once click

$("selectorbyclassorbyIDorbyName").click(function () {
  $("selectorbyclassorbyIDorbyName").attr("disabled", true).delay(2000).attr("disabled", false);
});

select the button and by its id or text or class ... it just disables after 1st click and enables after 20 Milli sec

Works very well for post backs n place it in Master page, applies to all buttons without calling implicitly like onclientClick

Hadoop MapReduce: Strange Result when Storing Previous Value in Memory in a Reduce Class (Java)

It is very inefficient to store all values in memory, so the objects are reused and loaded one at a time. See this other SO question for a good explanation. Summary:

[...] when looping through the Iterable value list, each Object instance is re-used, so it only keeps one instance around at a given time.

Having Django serve downloadable files

For the "best of both worlds" you could combine S.Lott's solution with the xsendfile module: django generates the path to the file (or the file itself), but the actual file serving is handled by Apache/Lighttpd. Once you've set up mod_xsendfile, integrating with your view takes a few lines of code:

from django.utils.encoding import smart_str

response = HttpResponse(mimetype='application/force-download') # mimetype is replaced by content_type for django 1.7
response['Content-Disposition'] = 'attachment; filename=%s' % smart_str(file_name)
response['X-Sendfile'] = smart_str(path_to_file)
# It's usually a good idea to set the 'Content-Length' header too.
# You can also set any other required headers: Cache-Control, etc.
return response

Of course, this will only work if you have control over your server, or your hosting company has mod_xsendfile already set up.

EDIT:

mimetype is replaced by content_type for django 1.7

response = HttpResponse(content_type='application/force-download')  

EDIT: For nginx check this, it uses X-Accel-Redirect instead of apache X-Sendfile header.

"Use the new keyword if hiding was intended" warning

In the code below, Class A implements the interface IShow and implements its method ShowData. Class B inherits Class A. In order to use ShowData method in Class B, we have to use keyword new in the ShowData method in order to hide the base class Class A method and use override keyword in order to extend the method.

interface IShow
{
    protected void ShowData();
}

class A : IShow
{
    protected void ShowData()
    {
        Console.WriteLine("This is Class A");
    }
}

class B : A
{
    protected new void ShowData()
    {
        Console.WriteLine("This is Class B");
    }
}

Remove a cookie

To reliably delete a cookie it's not enough to set it to expire anytime in the past, as computed by your PHP server. This is because client computers can and often do have times which differ from that of your server.

The best practice is to overwrite the current cookie with a blank cookie which expires one second in the future after the epoch (1 January 1970 00:00:00 UTC), as so:

setcookie("hello", "", 1);

CSS force image resize and keep aspect ratio

https://jsfiddle.net/sot2qgj6/3/

Here is the answer if you want to put image with fixed percentage of width, but not fixed pixel of width.

And this will be useful when dealing with different size of screen.

The tricks are

  1. Using padding-top to set the height from width.
  2. Using position: absolute to put image in the padding space.
  3. Using max-height and max-width to make sure the image will not over the parent element.
  4. using display:block and margin: auto to center the image.

I've also comment most of the tricks inside the fiddle.


I also find some other ways to make this happen. There will be no real image in html, so I personly perfer the top answer when I need "img" element in html.

simple css by using background http://jsfiddle.net/4660s79h/2/

background-image with word on top http://jsfiddle.net/4660s79h/1/

the concept to use position absolute is from here http://www.w3schools.com/howto/howto_css_aspect_ratio.asp

Combine two tables that have no common fields

try:

select * from table 1 left join table2 as t on 1 = 1;

This will bring all the columns from both the table.

Convert String value format of YYYYMMDDHHMMSS to C# DateTime

class Program
{
    static void Main(string[] args)
    {

        int transactionDate = 20201010;
        int? transactionTime = 210000;

        var agreementDate = DateTime.Today;
        var previousDate = agreementDate.AddDays(-1);

        var agreementHour = 22;
        var agreementMinute = 0;
        var agreementSecond = 0;

        var startDate = new DateTime(previousDate.Year, previousDate.Month, previousDate.Day, agreementHour, agreementMinute, agreementSecond);
        var endDate = new DateTime(agreementDate.Year, agreementDate.Month, agreementDate.Day, agreementHour, agreementMinute, agreementSecond);

        DateTime selectedDate = Convert.ToDateTime(transactionDate.ToString().Substring(6, 2) + "/" + transactionDate.ToString().Substring(4, 2) + "/" + transactionDate.ToString().Substring(0, 4) + " " + string.Format("{0:00:00:00}", transactionTime));

        Console.WriteLine("Selected Date : " + selectedDate.ToString());
        Console.WriteLine("Start Date : " + startDate.ToString());
        Console.WriteLine("End Date : " + endDate.ToString());

        if (selectedDate > startDate && selectedDate <= endDate)
            Console.WriteLine("Between two dates..");
        else if (selectedDate <= startDate)
            Console.WriteLine("Less than or equal to the start date!");
        else if (selectedDate > endDate)
            Console.WriteLine("Greater than end date!");
        else
            Console.WriteLine("Out of date ranges!");
    }
}

PHP - Insert date into mysql

How to debug SQL queries when you stuck

Print you query and run it directly in mysql or phpMyAdmin

$date = "2012-08-06";
$query= "INSERT INTO data_table (title, date_of_event) 
             VALUES('". $_POST['post_title'] ."',
                    '". $date ."')";
echo $query;
mysql_query($query) or die(mysql_error()); 

that way you can make sure that the problem is not in your PHP-script, but in your SQL-query

How to submit questions on SQ-queries

Make sure that you provided enough closure

  • Table schema
  • Query
  • Error message is any

Is it possible to start a shell session in a running container (without ssh)

There are two ways.

With attach

$ sudo docker attach 665b4a1e17b6 #by ID

With exec

$ sudo docker exec - -t 665b4a1e17b6 #by ID

Formatting a Date String in React Native

There is no need to include a bulky library such as Moment.js to fix such a simple issue.

The issue you are facing is not with formatting, but with parsing.

As John Shammas mentions in another answer, the Date constructor (and Date.parse) are picky about the input. Your 2016-01-04 10:34:23 may work in one JavaScript implementation, but not necessarily in the other.

According to the specification of ECMAScript 5.1, Date.parse supports (a simplification of) ISO 8601. That's good news, because your date is already very ISO 8601-like.

All you have to do is change the input format just a little. Swap the space for a T: 2016-01-04T10:34:23; and optionally add a time zone (2016-01-04T10:34:23+01:00), otherwise UTC is assumed.

Tools: replace not replacing in Android manifest

You can replace those in your Manifest application tag:

<application
    ...
    tools:replace="android:label, android:icon, android:theme"/>

and will work for you.

Explanation

Using such a dependency/library in your gradle file which has those labels in its Manifest's application tag may produce this problem and replacing them in your Manifest is the solution.

Vue.js: Conditional class style binding

Use the object syntax.

v-bind:class="{'fa-checkbox-marked': content['cravings'],  'fa-checkbox-blank-outline': !content['cravings']}"

When the object gets more complicated, extract it into a method.

v-bind:class="getClass()"

methods:{
    getClass(){
        return {
            'fa-checkbox-marked': this.content['cravings'],  
            'fa-checkbox-blank-outline': !this.content['cravings']}
    }
}

Finally, you could make this work for any content property like this.

v-bind:class="getClass('cravings')"

methods:{
  getClass(property){
    return {
      'fa-checkbox-marked': this.content[property],
      'fa-checkbox-blank-outline': !this.content[property]
    }
  }
}

Pandas: Return Hour from Datetime Column Directly

You can try this:

sales['time_hour'] = pd.to_datetime(sales['timestamp']).dt.hour

TypeScript sorting an array

Numbers

When sorting numbers, you can use the compact comparison:

var numericArray: number[] = [2, 3, 4, 1, 5, 8, 11];

var sortedArray: number[] = numericArray.sort((n1,n2) => n1 - n2);

i.e. - rather than <.

Other Types

If you are comparing anything else, you'll need to convert the comparison into a number.

var stringArray: string[] = ['AB', 'Z', 'A', 'AC'];

var sortedArray: string[] = stringArray.sort((n1,n2) => {
    if (n1 > n2) {
        return 1;
    }

    if (n1 < n2) {
        return -1;
    }

    return 0;
});

Objects

For objects, you can sort based on a property, bear in mind the above information about being able to short-hand number types. The below example works irrespective of the type.

var objectArray: { age: number; }[] = [{ age: 10}, { age: 1 }, {age: 5}];

var sortedArray: { age: number; }[] = objectArray.sort((n1,n2) => {
    if (n1.age > n2.age) {
        return 1;
    }

    if (n1.age < n2.age) {
        return -1;
    }

    return 0;
});

How does delete[] know it's an array?

Hey ho well it depends of what you allocating with new[] expression when you allocate array of build in types or class / structure and you don't provide your constructor and destructor the operator will treat it as a size "sizeof(object)*numObjects" rather than object array therefore in this case number of allocated objects will not be stored anywhere, however if you allocate object array and you provide constructor and destructor in your object than behavior change, new expression will allocate 4 bytes more and store number of objects in first 4 bytes so the destructor for each one of them can be called and therefore new[] expression will return pointer shifted by 4 bytes forward, than when the memory is returned the delete[] expression will call a function template first, iterate through array of objects and call destructor for each one of them. I've created this simple code witch overloads new[] and delete[] expressions and provides a template function to deallocate memory and call destructor for each object if needed:

// overloaded new expression 
void* operator new[]( size_t size )
{
    // allocate 4 bytes more see comment below 
    int* ptr = (int*)malloc( size + 4 );

    // set value stored at address to 0 
    // and shift pointer by 4 bytes to avoid situation that
    // might arise where two memory blocks 
    // are adjacent and non-zero
    *ptr = 0;
    ++ptr; 

    return ptr;
}
//////////////////////////////////////////

// overloaded delete expression 
void static operator delete[]( void* ptr )
{
    // decrement value of pointer to get the
    // "Real Pointer Value"
    int* realPtr = (int*)ptr;
    --realPtr;

    free( realPtr );
}
//////////////////////////////////////////

// Template used to call destructor if needed 
// and call appropriate delete 
template<class T>
void Deallocate( T* ptr )
{
    int* instanceCount = (int*)ptr;
    --instanceCount;

    if(*instanceCount > 0) // if larger than 0 array is being deleted
    {
        // call destructor for each object
        for(int i = 0; i < *instanceCount; i++)
        {
            ptr[i].~T();
        }
        // call delete passing instance count witch points
        // to begin of array memory 
        ::operator delete[]( instanceCount );
    }
    else
    {
        // single instance deleted call destructor
        // and delete passing ptr
        ptr->~T();
        ::operator delete[]( ptr );
    }
}

// Replace calls to new and delete
#define MyNew ::new
#define MyDelete(ptr) Deallocate(ptr)

// structure with constructor/ destructor
struct StructureOne
{
    StructureOne():
    someInt(0)
    {}
    ~StructureOne() 
    {
        someInt = 0;
    }

    int someInt;
};
//////////////////////////////

// structure without constructor/ destructor
struct StructureTwo
{
    int someInt;
};
//////////////////////////////


void main(void)
{
    const unsigned int numElements = 30;

    StructureOne* structOne = nullptr;
    StructureTwo* structTwo = nullptr;
    int* basicType = nullptr;
    size_t ArraySize = 0;

/**********************************************************************/
    // basic type array 

    // place break point here and in new expression
    // check size and compare it with size passed 
    // in to new expression size will be the same
    ArraySize = sizeof( int ) * numElements;

    // this will be treated as size rather than object array as there is no 
    // constructor and destructor. value assigned to basicType pointer
    // will be the same as value of "++ptr" in new expression
    basicType = MyNew int[numElements];

    // Place break point in template function to see the behavior
    // destructors will not be called and it will be treated as 
    // single instance of size equal to "sizeof( int ) * numElements"
    MyDelete( basicType );

/**********************************************************************/
    // structure without constructor and destructor array 

    // behavior will be the same as with basic type 

    // place break point here and in new expression
    // check size and compare it with size passed 
    // in to new expression size will be the same
    ArraySize = sizeof( StructureTwo ) * numElements;

    // this will be treated as size rather than object array as there is no 
    // constructor and destructor value assigned to structTwo pointer
    // will be the same as value of "++ptr" in new expression
    structTwo = MyNew StructureTwo[numElements]; 

    // Place break point in template function to see the behavior
    // destructors will not be called and it will be treated as 
    // single instance of size equal to "sizeof( StructureTwo ) * numElements"
    MyDelete( structTwo );

/**********************************************************************/
    // structure with constructor and destructor array 

    // place break point check size and compare it with size passed in
    // new expression size in expression will be larger by 4 bytes
    ArraySize = sizeof( StructureOne ) * numElements;

    // value assigned to "structOne pointer" will be different 
    // of "++ptr" in new expression  "shifted by another 4 bytes"
    structOne = MyNew StructureOne[numElements];

    // Place break point in template function to see the behavior
    // destructors will be called for each array object 
    MyDelete( structOne );
}
///////////////////////////////////////////

How do I list all loaded assemblies?

Using Visual Studio

  1. Attach a debugger to the process (e.g. start with debugging or Debug > Attach to process)
  2. While debugging, show the Modules window (Debug > Windows > Modules)

This gives details about each assembly, app domain and has a few options to load symbols (i.e. pdb files that contain debug information).

enter image description here

Using Process Explorer

If you want an external tool you can use the Process Explorer (freeware, published by Microsoft)

Click on a process and it will show a list with all the assemblies used. The tool is pretty good as it shows other information such as file handles etc.

Programmatically

Check this SO question that explains how to do it.

Using Page_Load and Page_PreRender in ASP.Net

Well a big requirement to implement PreRender as opposed to Load is the need to work with the controls on the page. On Page_Load, the controls are not rendered, and therefore cannot be referenced.

Download old version of package with NuGet

By using the Nuget Package Manager UI as mentioned above it helps to uninstall the nuget package first. I always have problems when going back on a nuget package version if I don't uninstall first. Some references are not cleaned properly. So I suggest the following workflow when installing an old nuget package through the Nuget Package Manager:

  1. Selected your nuget server / source
  2. Find and select the nuget package your want to install an older version
  3. Uninstall current version
  4. Click on the install drop-down > Select older version > Click Install

enter image description here

Good Luck :)

Python: slicing a multi-dimensional array

If you use numpy, this is easy:

slice = arr[:2,:2]

or if you want the 0's,

slice = arr[0:2,0:2]

You'll get the same result.

*note that slice is actually the name of a builtin-type. Generally, I would advise giving your object a different "name".


Another way, if you're working with lists of lists*:

slice = [arr[i][0:2] for i in range(0,2)]

(Note that the 0's here are unnecessary: [arr[i][:2] for i in range(2)] would also work.).

What I did here is that I take each desired row 1 at a time (arr[i]). I then slice the columns I want out of that row and add it to the list that I'm building.

If you naively try: arr[0:2] You get the first 2 rows which if you then slice again arr[0:2][0:2], you're just slicing the first two rows over again.

*This actually works for numpy arrays too, but it will be slow compared to the "native" solution I posted above.

Float right and position absolute doesn't work together

I was able to absolutely position a right-floated element with one layer of nesting and a tricky margin:

_x000D_
_x000D_
function test() {_x000D_
  document.getElementById("box").classList.toggle("hide");_x000D_
}
_x000D_
.right {_x000D_
  float:right;_x000D_
}_x000D_
#box {_x000D_
  position:absolute; background:#feb;_x000D_
  width:20em; margin-left:-20em; padding:1ex;_x000D_
}_x000D_
#box.hide {_x000D_
  display:none;_x000D_
}
_x000D_
<div>_x000D_
  <div class="right">_x000D_
    <button onclick="test()">box</button>_x000D_
    <div id="box">Lorem ipsum dolor sit amet, consectetur adipiscing elit,_x000D_
      sed do eiusmod tempor incididunt ut labore et dolore magna aliqua._x000D_
      Ut enim ad minim veniam, quis nostrud exercitation ullamco laboris_x000D_
      nisi ut aliquip ex ea commodo consequat._x000D_
    </div>_x000D_
  </div>_x000D_
  <p>_x000D_
    Lorem ipsum dolor sit amet, consectetur adipiscing elit,_x000D_
    sed do eiusmod tempor incididunt ut labore et dolore magna aliqua._x000D_
    Ut enim ad minim veniam, quis nostrud exercitation ullamco laboris_x000D_
    nisi ut aliquip ex ea commodo consequat._x000D_
  </p>_x000D_
</div>
_x000D_
_x000D_
_x000D_

I decided to make this toggleable so you can see how it does not affect the flow of the surrounding text (run it and press the button to show/hide the floated absolute box).

Get data from JSON file with PHP

Get the content of the JSON file using file_get_contents():

$str = file_get_contents('http://example.com/example.json/');

Now decode the JSON using json_decode():

$json = json_decode($str, true); // decode the JSON into an associative array

You have an associative array containing all the information. To figure out how to access the values you need, you can do the following:

echo '<pre>' . print_r($json, true) . '</pre>';

This will print out the contents of the array in a nice readable format. Note that the second parameter is set to true in order to let print_r() know that the output should be returned (rather than just printed to screen). Then, you access the elements you want, like so:

$temperatureMin = $json['daily']['data'][0]['temperatureMin'];
$temperatureMax = $json['daily']['data'][0]['temperatureMax'];

Or loop through the array however you wish:

foreach ($json['daily']['data'] as $field => $value) {
    // Use $field and $value here
}

Demo!

PHP: How to get current time in hour:minute:second?

Anytime you have a question about a particular function in PHP, the easiest way to get quick answers is by visiting php.net, which has great documentation on all of the language's capabilities.

Looking up a function is easy, just visit http://php.net/<function name> and it will forward you to the appropriate place. For the date function, we'll visit http://php.net/date.

We immediately learn a couple things about this function by examining its signature:

string date ( string $format [, int $timestamp = time() ] )

First, it returns a string. That's what the first string in the above code means. Secondly, the first parameter is expected to be a string containing the format. There is an optional second parameter for passing in your own timestamp (to construct strings from some time other than now).

date("d-m-Y") // produces something like 03-12-2012

In this code, d represents the day of the month (with a leading 0 is necessary). m represents the month, again with a leading zero if necessary. And Y represents the full 4-digit year. All of these are documented in the aforementioned link.

To satisfy your request of getting the hours, minutes, and seconds, we need to give a quick look at the documentation to see which characters represents those particular units of time. When we do that, we find the following:

h   12-hour format of an hour with leading zeros    01 through 12
i   Minutes with leading zeros                      00 to 59
s   Seconds, with leading zeros                     00 through 59

With this in mind, we can no create a new format string:

date("d-m-Y h:i:s"); // produces something like 03-12-2012 03:29:13

Hope this is helpful, and I hope you find the documentation has benefiting to your development as I have to mine.

403 Forbidden vs 401 Unauthorized HTTP responses

See RFC2616:

401 Unauthorized:

If the request already included Authorization credentials, then the 401 response indicates that authorization has been refused for those credentials.

403 Forbidden:

The server understood the request, but is refusing to fulfill it.

Update

From your use case, it appears that the user is not authenticated. I would return 401.


Edit: RFC2616 is obsolete, see RFC7231 and RFC7235.

Best way to access a control on another form in Windows Forms?

I would handle this in the parent form. You can notify the other form that it needs to modify itself through an event.

header location not working in my php code

Try adding ob_start(); at the top of the code i.e. before the include statement.

Iterate through a C array

It depends. If it's a dynamically allocated array, that is, you created it calling malloc, then as others suggest you must either save the size of the array/number of elements somewhere or have a sentinel (a struct with a special value, that will be the last one).

If it's a static array, you can sizeof it's size/the size of one element. For example:

int array[10], array_size;
...
array_size = sizeof(array)/sizeof(int);

Note that, unless it's global, this only works in the scope where you initialized the array, because if you past it to another function it gets decayed to a pointer.

Hope it helps.

How to select ALL children (in any level) from a parent in jQuery?

Use jQuery.find() to find children more than one level deep.

The .find() and .children() methods are similar, except that the latter only travels a single level down the DOM tree.

$('#google_translate_element').find('*').unbind('click');

You need the '*' in find():

Unlike in the rest of the tree traversal methods, the selector expression is required in a call to .find(). If we need to retrieve all of the descendant elements, we can pass in the universal selector '*' to accomplish this.

How to monitor the memory usage of Node.js?

node-memwatch : detect and find memory leaks in Node.JS code. Check this tutorial Tracking Down Memory Leaks in Node.js

Javascript | Set all values of an array

Actually, you can use this perfect approach:

let arr = Array.apply(null, Array(5)).map(() => 0);
// [0, 0, 0, 0, 0]

This code will create array and fill it with zeroes. Or just:

let arr = new Array(5).fill(0)

Named placeholders in string formatting

I am the author of a small library that does exactly what you want:

Student student = new Student("Andrei", 30, "Male");

String studStr = template("#{id}\tName: #{st.getName}, Age: #{st.getAge}, Gender: #{st.getGender}")
                    .arg("id", 10)
                    .arg("st", student)
                    .format();
System.out.println(studStr);

Or you can chain the arguments:

String result = template("#{x} + #{y} = #{z}")
                    .args("x", 5, "y", 10, "z", 15)
                    .format();
System.out.println(result);

// Output: "5 + 10 = 15"

SSL_connect: SSL_ERROR_SYSCALL in connection to github.com:443

The problem for me seems to have been how the user has been setup on my local machine to. Using the command
git push -u origin master
was causing the error. Removing the switch -u to have
git push origin master
solved it for me. It can be scary to imagine how user setup can result in an error related to LibreSSL.

How to retrieve data from a SQL Server database in C#?

To retrieve data from database:

private SqlConnection Conn;
 private void CreateConnection()
 {
    string ConnStr =
    ConfigurationManager.ConnectionStrings["ConnStr"].ConnectionString;
    Conn = new SqlConnection(ConnStr);
 }
 public DataTable getData()
 {
 CreateConnection();
    string SqlString = "SELECT * FROM TableName WHERE SomeID = @SomeID;";
    SqlDataAdapter sda = new SqlDataAdapter(SqlString, Conn);
    DataTable dt = new DataTable();
    try
    {
        Conn.Open();
        sda.Fill(dt);
    }
    catch (SqlException se)
    {
        DBErLog.DbServLog(se, se.ToString());
    }
    finally
    {
        Conn.Close();
    }
    return dt;
}

How to add a class to a given element?

document.getElementById('some_id').className+='  someclassname'

OR:

document.getElementById('some_id').classList.add('someclassname')

First approach helped in adding the class when second approach didn't work.
Don't forget to keep a space in front of the ' someclassname' in the first approach.

For removal you can use:

document.getElementById('some_id').classList.remove('someclassname')

Difference between long and int data types

From this reference:

An int was originally intended to be the "natural" word size of the processor. Many modern processors can handle different word sizes with equal ease.

Also, this bit:

On many (but not all) C and C++ implementations, a long is larger than an int. Today's most popular desktop platforms, such as Windows and Linux, run primarily on 32 bit processors and most compilers for these platforms use a 32 bit int which has the same size and representation as a long.

Command output redirect to file and terminal

In case somebody needs to append the output and not overriding, it is possible to use "-a" or "--append" option of "tee" command :

ls 2>&1 | tee -a /tmp/ls.txt
ls 2>&1 | tee --append /tmp/ls.txt